• Shuffle
    Toggle On
    Toggle Off
  • Alphabetize
    Toggle On
    Toggle Off
  • Front First
    Toggle On
    Toggle Off
  • Both Sides
    Toggle On
    Toggle Off
  • Read
    Toggle On
    Toggle Off
Reading...
Front

Card Range To Study

through

image

Play button

image

Play button

image

Progress

1/59

Click to flip

Use LEFT and RIGHT arrow keys to navigate between flashcards;

Use UP and DOWN arrow keys to flip the card;

H to show hint;

A reads text to speech;

59 Cards in this Set

  • Front
  • Back
A 29-year-old man presents to the emergency department because he has experienced headache, fatigue, malaise, and fevers for the last 5 days. He states these symptoms have been worsening over time and in the last 2 days, have been accompanied by light aversion and a stiff neck. He was diagnosed with human immunodeficiency virus (HIV) 9 months ago and has not taken antiretroviral therapy. Physical examination reveals a body temperature of 39.1°C (102.3°F) and an inability to passively flex the neck so that the chin touches the chest. A lumbar puncture is performed and examination of the cerebrospinal fluid (CSF) sediment is pictured. What is the most appropriate therapy for this patient?
budding yeast with thick capsule

Answer Choices Correct AnswerCorrect answer Your AnswerYour answer
A. Amphotericin B
B. Amphotericin B and flucytosine
C. Amphotericin B, flucytosine, and steroids
D. Fluconazole
E. Fluconazole and flucytosine
F. Fluconazole, flucytosine, and steroids
G. Pyrimethamine
H. Pyrimethamine and sulfadiazine
I. Trimethoprim-sulfamethoxazole
J. Trimethoprim-sulfamethoxazole (TMP-SMX) and steroids
Option B (Amphotericin B and flucytosine) is correct. This patient has cryptococcosis, caused by Cryptococcus neoformans. The budding yeast with thick capsule can be seen on India ink examination of the cerebrospinal fluid (CSF). Cryptococcal antigen analysis should also be performed on the CSF to confirm the diagnosis. Treatment has been demonstrated in randomized controlled trials to be most effective when Amphotericin B is combined with flucytosine. Flucytosine enters fungal cells, is converted to fluorouracil that can compete with uracil and cause disruption of RNA synthesis.

Option A (Amphotericin B) is incorrect. In trials comparing only amphotericin B to fluconazole, there was no significant overall difference. However, when flucytosine was added, there was a greater survival for this combination therapy. Thus, amphotericin B should not be used alone.

Option C (Amphotericin B, flucytosine, and steroids) is incorrect. Steroids do not have a role in the management of cryptococcosis.

Option D (Fluconazole) is incorrect. Fluconazole has been shown to be effective, but not as effective as the combination of amphotericin B and flucytosine. Fluconazole is associated with bone marrow suppression.

Option E (Fluconazole and flucytosine) is incorrect. This combination has not be used in the management of cryptococcosis.

Option F (Fluconazole, flucytosine, and steroids) is incorrect. Steroids do not have a role in the management of cryptococcosis.

Option G (Pyrimethamine) is incorrect. This is inappropriate therapy for any condition.

Option H (Pyrimethamine and sulfadiazine) is incorrect. This is the appropriate therapy for toxoplasmosis, which presents in human immunodeficiency virus (HIV) patients that have a CD4+ count below 100. There are ring-enhancing lesions on neuroimaging.

Option I (Trimethoprim-sulfamethoxazole) is incorrect. This is appropriate for the prevention of toxoplasmosis and Pneumocystis carinii pneumonia (PCP). It is also therapeutic against PCP infection.

Option J (Trimethoprim-sulfamethoxazole [TMP-SMX] and steroids) is incorrect. Steroids should be added to the treatment of Pneumocystis carinii pneumonia (PCP) if there is a PaO2 less than 70 or a A-a gradient greater than 35.

High-yield Hit 1
Figure 75-8 Cryptococcus neoformans India ink preparation demonstrating the large capsule surrounding budding yeast cells (magnification 1000×).

From Medical Microbiology 5E by Murray et al
High-yield Hit 2
Treatment
Table 75-7. Sensitivity of Antigen Detection, India Ink Microscopy, and Culture of Cerebral Spinal Fluid in the Diagnosis of Cryptococcal Meningitis
Test % Sensitivity
AIDS Patients Non-AIDS Patients
Antigen 100 86-95
India ink 82 50
Culture 100 90

Adapted from Viviani MA, Tortorano AM, Ajello L: Cryptococcus. In Anaissie EJ, McGinnis MR, Pfaller MA, editors: Clinical mycology, New York, 2003, Churchill Livingstone.
Cryptococcal meningitis and other disseminated forms of cryptococcosis are universally fatal if left untreated. All patients should receive amphotericin B plus flucytosine acutely for 2 weeks (induction therapy), followed by 8-week consolidation with either oral fluconazole (preferred) or itraconazole. AIDS patients generally require lifelong maintenance therapy with either fluconazole or itraconazole. In patients who do not have AIDS, treatment can be discontinued after the consolidation therapy; however, relapse may be seen in up to 26% of these patients within 3 to 6 months after discontinuation of therapy. Thus a prolonged consolidation treatment with an azole for up to 1 year may be advisable even in patients without AIDS.
Treatment of these patients should be followed up both clinically and mycologically. Mycologic follow-up requires repeat lumbar puncture to be performed (1) at the end of the 2-week induction therapy to ensure sterilization of the CSF; (2) at the end of the consolidation therapy; and (3) whenever indicated by a change in clinical status during follow-up. CSF samples collected during follow-up must undergo culture. Determination of CSF protein, glucose, cell count, and cryptococcal antigen titer are helpful in assessing the response to therapy but are not highly predictive of outcome. Failure to sterilize the CSF by day 14 of therapy is indicative of a much higher probability that the consolidation therapy will fail.

From Medical Microbiology 5E by Murray et al
Parents of an 8-year-old boy bring their child to their community health clinic for symptoms that began with a fever 4 days ago. Since then, the child has become weak and lethargic and has developed a poor appetite, red irritated–appearing eyes, a cough, and a red runny nose. Past medical history is noncontributory. The family emigrated to the United States from West Africa 1 week ago. On exam, the child has a nonproductive cough. Vital signs are temperature: 38.9°C (102°F), blood pressure (BP): 110/65, heart rate (HR): 90 beats/min, respiratory rate (RR): 15 breaths/min. There is conjunctival and nasal mucosal inflammation with a clear watery discharge. Inspection of the buccal mucosa shows bluish-grey speckles on a red base opposite the second molars. The remainder of the exam is normal. What is associated with the highest increased morbidity risk for this patient's diagnosis?

Answer Choices Correct AnswerCorrect answer Your AnswerYour answer
A. Appearance of a desquamating rash
B. Lymphadenopathy
C. Patient's age
D. Patient's sex
E. Vitamin A deficiency
Option E (Vitamin A deficiency) is correct. This is rubeola, or classical “10-day measles” in the prerash, prodromal phase characterized by malaise, fever, anorexia, conjunctivitis, cough, and coryza. The buccal exanthema represents the pathognomonic Koplik's spots. The World Health Organization (WHO) recommends vitamin A administration to all children with measles in communities where vitamin A deficiency is a recognized problem and where the measles virus (MV)-related mortality rate exceeds 1%. Of note, low serum concentrations of vitamin A are found in children with severe measles in the United States. Vitamin A supplementation during acute measles significantly reduces risks of morbidity and mortality.

Option A (Appearance of a desquamating rash) is incorrect. The appearance of a desquamating rash is the typical progression of the disease. The rash may not appear in immunosuppressed individuals, and thus its absence may imply an increased morbidity. Immunosuppressed individuals are predisposed to severe bacterial infection, particularly bronchopneumonia, a major cause of measles-related mortality among younger children.

Option B (Lymphadenopathy) is incorrect. Generalized lymphadenopathy is common in measles and is not a recognized association with increased disease morbidity.

Option C (Patient's age) is incorrect. Case-fatality rates are higher among children younger than age 5 years. Highest fatality rates are among infants aged 4 to12 months

Option D (Patient's sex) is incorrect. Excess mortality following acute measles has been observed among females at all ages

High-yield Hit 1
Treatment
There is no specific therapy for measles. Routine supportive care includes maintaining adequate hydration and antipyretics. Photophobia is intensified by strong light, which should be avoided. IV ribavirin may be beneficial in severe infections.
High-dose vitamin A supplementation improves the outcome in infected malnourished infants and should be considered for populations at risk for severe complications, including infants 6 months to 2 years old requiring hospitalization, HIV-infected infants, and infants from endemic areas in developing countries.

From Nelson Essentials of Pediatrics 5E by Kliegman et al
treatment of Pneumocystis carinii pneumonia (PCP) if there is a PaO2 less than 70 or a A-a gradient greater than 35.
Trimethoprim-sulfamethoxazole [TMP-SMX] and steroids

Steroids should be added to the treatment of Pneumocystis carinii pneumonia (PCP) if there is a PaO2 less than 70 or a A-a gradient greater than 35.
A 36-year-old man presents to the emergency department with a 12-hour history of headache, fever, chills, and photophobia. He was previously well and does not take any regular medications. On arrival, his vital signs are blood pressure (BP), 123/78 mm Hg; pulse, 104 beats per minute; temperature, 39.2°C (102.5°F); respiration rate, 25 breaths per minute. There is nuchal rigidity on neck flexion. Passive knee extension elicits neck pain. What is the most likely cause of these findings?

Answer Choices Correct AnswerCorrect answer Your AnswerYour answer
A. Haemophilus influenzae
B. Listeria monocytogenes
C. Neisseria meningitidis
D. Streptococcus agalactiae
E. Streptococcus pneumoniae
Option E (Streptococcus pneumoniae) is correct. This patient has signs and symptoms that lead to a presumptive diagnosis of meningitis. The most common cause of bacterial meningitis in adults is Streptococcus pneumoniae.

Option A (Haemophilus influenzae) is incorrect. Haemophilus influenzae was the most common cause of meningitis in children until immunization to type b H. influenzae. It is now a relatively infrequent cause.

Option B (Listeria monocytogenes) is incorrect. Listeria monocytogenesis the third most common cause of meningitis is neonates. It is much more common in immunosuppressed adults, the elderly, and pregnant women.

Option C (Neisseria meningitidis) is incorrect. This is the second most common cause of meningitis in adults and often presents with a classic petechial/purpuric rash. It is the most common cause of meningitis in children.

Option D (Streptococcus agalactiae) is incorrect. This is the most common cause of bacterial meningitis is neonates.

High-yield Hit 1
Clinical Presentation
Patients with bacterial meningitis may present with fever, headache, lethargy, confusion, irritability, and stiff neck. There are three principal modes of onset. About 25% of cases begin abruptly with fulminant illness; mortality is high in this setting. More often, meningeal symptoms progress over 1 to 7 days. Thirdly, meningitis may superimpose itself on 1 to 3 weeks of an upper respiratory-type illness; diagnosis is most difficult in this group. Occasionally, no more than a single additional neurologic symptom or sign hints at disease more serious than a routine upper respiratory tract infection. Stiff neck is absent in roughly half of all patients with meningitis, notably in the very young, the old, and the comatose. A petechial or purpuric rash is found in one half of patients with meningococcemia; although not pathognomonic, palpable purpura is very suggestive of N. meningitidis infection. About 20% of patients with acute bacterial meningitis have seizures, and a similar fraction have focal neurologic findings.

From Cecil Essentials of Medicine 6E by Andreoli et al
High-yield Hit 2
Epidemiology
Three fourths of cases of acute bacterial meningitis occur before the age of 15 years. Neisseria meningitidis causes sporadic disease or epidemics in closed populations. Most cases occur in winter and spring and involve children younger than 5 years of age. Haemophilus influenzae meningitis is even more selectively a disease of childhood, with most cases developing by the age of 10 years. Infections are sporadic, although secondary cases may occur in close contacts. The incidence of H. influenzae meningitis has significantly declined with the widespread use of effective conjugated vaccines. In contrast, pneumococcal meningitis is a disease seen in all age groups and increasing generally in pediatric groups. In addition, pneumococcus is becoming increasingly resistant to penicillin, making therapy much more challenging (see later discussion). Recent clinical series of hospitalized adults show a relative frequency of 40 to 55% pneumococcal, 3 to 13% meningococcal, 10 to 13% listerial, and 4 to 8% H. influenzae for community-acquired meningitis.

From Cecil Essentials of Medicine 6E by Andreoli et al
This is the second most common cause of meningitis in adults and often presents with a classic petechial/purpuric rash. It is the most common cause of meningitis in children.
Neisseria meningitidis)
The most common cause of bacterial meningitis in adults
Streptococcus pneumoniae)
the most common cause of bacterial meningitis is neonates
Streptococcus agalactiae
A 40-year-old woman complains to her physician of a gradual onset of pain in her neck and increasing generalized weakness and fatigue. The pain began 1 month ago. She reports that she had the flu several months ago and is now having difficulty swallowing. Physical examination shows a nervous-appearing woman with a temperature of 28.3°C (101°F) and heart rate of 100 beats/minute. The anterior cervical region of the neck is tender to palpation. Her laboratory tests show low serum thyroid-stimulating hormone (TSH), a serum-free thyroxine of 11.5 μg/dL, and an elevated erythrocyte sedimentation rate. A radioactive iodine uptake (RAIU) test shows reduced uptake. The most likely diagnosis is

Answer Choices Correct AnswerCorrect answer Your AnswerYour answer
A. Autonomously functioning thyroid nodule
B. Graves disease
C. Subacute thyroiditis
D. Toxic multinodular goiter
E. TSH-secreting pituitary tumor
Option C (Subacute thyroiditis) is correct. This patient is experiencing symptoms of thyrotoxicosis, but the challenge is to determine why. In the RAIU test, radioactive iodine is administered to the patient, and the percentage taken up by the thyroid gland is measured 24 hours later. Of the possibilities shown, only thyroiditis would yield a low RAIU test.

Option A (Autonomously functioning thyroid nodule) is incorrect. The RAIU test for an autonomously functioning nodule would show increased uptake.

Option B (Graves disease) is incorrect. The RAIU test for Graves disease would show increased uptake.

Option D (Toxic multinodular goiter) is incorrect. The RAIU test for toxic multinodular goiter would show increased uptake.

Option E (TSH-secreting pituitary tumor) is incorrect. The RAIU test for a TSH-secreting pituitary tumor would show increased uptake, but also would result in a high TSH with high thyroxine.

High-yield Hit 1
3. Describe the four stages of subacute thyroiditis.

* Stage I: Patients have a painful (unilateral or bilateral) tender thyroid and may have systemic symptoms (fatigue, malaise, fever). Inflammatory destruction of thyroid follicles allows release of excess thyroxine (T4) and triiodothyronine (T3) into the blood, and thyrotoxicosis may ensue.
* Stage II: A transitory period (several weeks) of euthyroidism occurs after the T4 is cleared from the body.
* Stage III: With severe disease, patients may become hypothyroid until the thyroid gland repairs itself.
* Stage IV: Euthyroid state returns.



Taken from Endocrine Secrets 4E by McDermott
High-yield Hit 2
4. Summarize the natural history of subacute thyroiditis.
Subacute thyroiditis is probably viral in origin. Histologically, the inflammation is granulomatous. Although patients almost always recover clinically, serum thyroglobulin levels remain elevated and intrathyroidal iodine content is low for many months (Fig. 36-1). Patients requiring steroids are more likely to become hypothyroid at a later time. Such findings suggest persistent subclinical abnormalities after an episode of subacute thyroiditis. About 2% of patients have a second episode many years later.


Taken from Endocrine Secrets 4E by McDermott
31-year-old man presents to the physician with a lesion on his penis. He states that the lesion has been present for the last week and is extremely painful. His previous medical history is unremarkable; he does not take any regular medications and has no known allergies. He recently immigrated from southern Africa, works as a general laborer, does not drink, and does not smoke. He is sexually active and has had eight female sexual partners in the last 12 months. The physical examination reveals a 1.5-cm (0.6-in), sharply defined ulcer. The base is covered with a yellow-gray purulent exudate. It bleeds when scraped. The Gram stain reveals gram-negative rods in a school of fish appearance.

Answer Choices Correct AnswerCorrect answer Your AnswerYour answer
A. Chancroid
B. Chlamydia
C. Condylomata acuminata
D. Gonorrhea
E. Granuloma inguinale
F. Herpes simplex
G. Lymphogranuloma venereum
H. Molluscum contagiosum
I. Syphilis
J. Trichomoniasis
Option A (Chancroid) is correct. This patient has chancroid, which is caused by Haemophilus ducreyi. It is an uncommon sexually transmitted disease in the United States, but quite common in Africa and Southeast Asia. It presents as a painful ulcer with ragged borders. Inguinal lymphadenitis affects approximately half of patients, and when they undergo liquefaction, they become buboes. The classic Gram-stain pattern is as small gram-negative rods in chains that resemble a school of fish.

High-yield Hit 1
Chancroid (soft chancre)
Chancroid is caused by Haemophilus ducreyi and is characterized by painful genital ulcers
Figure 21.19 Chancroid. Several irregular ulcers on the prepuce. (Courtesy of L Parish.)
Infection by the Gram-negative bacterium Haemophilus ducreyi is manifest as painful non-indurated genital ulcers and local lymphadenitis (Fig. 21.19). Note the difference between this and the chancre of primary syphilis, which is painless, but the ulcers may be confused with those of genital herpes, though they are usually larger and have a more ragged appearance. While the disease is endemic in some areas of the USA, cases generally tend to occur in distinct outbreaks. However, in Africa and Asia chancroid is the commonest cause of genital ulcers. Epidemiologic information is important because the diagnosis is usually clinical as the organism is difficult to grow in the laboratory. Chancroid may also be confused with donovanosis (see below).
Chancroid is diagnosed by microscopy and culture and treated with erythromycin, ceftriaxone or co-trimoxazole

From Medical Microbiology 3E by Mims et al
A 19-year-old woman presents to the public health clinic because she has developed multiple lesions over the inner thigh and labia majora. The physical examination reveals numerous 2 to 5 mm, firm, dome-shaped, pearly-white papules with an umbilicated center.

Answer Choices Correct AnswerCorrect answer Your AnswerYour answer
A. Chancroid
B. Chlamydia
C. Condylomata acuminata
D. Gonorrhea
E. Granuloma inguinale
F. Herpes simplex
G. Lymphogranuloma venereum
H. Molluscum contagiosum
I. Syphilis
J. Trichomoniasis
Option H (Molluscum contagiosum) is correct. This patient has molluscum contagiosum, as suggested by the characteristic appearance of the papules. The condition is caused by a poxvirus. The lesions typically resolve spontaneously within several months in immunocompetent individuals. Should the patient desire the lesions to be removed, a variety of mechanical methods are possible.

High-yield Hit 1
Molluscum Contagiosum
BASIC INFORMATION
DEFINITION
Viral infection characterized by discrete skin lesions with central umbilication (Fig. 1-150).
ICD-9CM CODES
078.0 Molluscum contagiosum
EPIDEMIOLOGY & DEMOGRAPHICS

* Molluscum contagiosum spreads by autoinoculation, scratching or touching a lesion.
* It usually occurs in young children. It is also common in sexually active adults and patients with HIV infection.
* Incubation period varies between 4 and 8 wk.
* Spontaneous resolution in immunocompetent patients can occur after several months.

PHYSICAL FINDINGS & CLINICAL PRESENTATION

* The individual lesion appears initially as a flesh-colored, firm, smooth-surfaced papule with subsequent central umbilication. Lesions are frequently grouped. The size of each lesion generally varies from 2 to 6 mm in diameter.
* Typical distribution in children involves the face, extremities, and trunk. Mucous membranes are spared.
* Distribution in adults generally involves pubic and genital areas.
* Erythema and scaling at the periphery of the lesions may be present as a result of scratching or hypersensitivity reaction.
* Lesions are not present on the palms and soles.

ETIOLOGY
Figure 1-150 Molluscum contagiosum. (From Rakel RE: Textbook of family practice, ed 6, Philadelphia, 2002, WB Saunders.)
Viral infection of epithelial cells caused by a pox virus
DIAGNOSIS
Diagnosis is usually established by the clinical appearance of the lesions (distribution and central umbilication). A magnifying lens can be used to observe the central umbilication. If necessary, the diagnosis can be confirmed by removing a typical lesion with a curette and examining the content on a slide after adding potassium hydroxide and gentle heating. Staining with toluidine blue will identify viral inclusions.
DIFFERENTIAL DIAGNOSIS

* Verruca plana (flat warts): no central umbilication, not dome shaped, irregular surface, can involve palms and soles
* Herpes simplex: lesions become rapidly umbilicated
* Varicella: blisters and vesicles are present
* Folliculitis: no central umbilication, presence of hair piercing the pustule or papule
* Cutaneous cryptococcosis in AIDS patients: budding yeasts will be present on cytologic examination of the lesions
* Basal cell carcinoma: multiple lesions are absent


From Ferri's Clinical Advisor 2006 by Ferri
A 66-year-old woman presents to the physician because she has noticed a change in her vision. She was diagnosed with active pulmonary tuberculosis (TB) 9 weeks ago. Her current medications include isoniazid, rifampin, ethambutol, pyrazinamide, and pyridoxine. On examination, there is a reduction in her visual acuity from 9 weeks ago when she initially began therapy. Her visual acuity was previously 20/20 in both eyes and is now 20/80 in both eyes. What drug is most likely responsible for these findings?

Answer Choices Correct AnswerCorrect answer Your AnswerYour answer
A. Ethambutol
B. Isoniazid
C. Pyrazinamide
D. Pyridoxine
E. Rifampin
Option A (Ethambutol) is correct. A complication of ethambutol is optic neuritis, which usually occurs at high doses but can also occur at standard doses. Red-green color vision disturbance is first noted, followed by a decrease in visual acuity. There should be regular monitoring of visual acuity during therapy.

Option B (Isoniazid) is incorrect. Isoniazid is associated most commonly with allergic skin eruptions, but is more seriously associated with hepatitis, vasculitis and peripheral neuropathy if pyridoxine is not co-administered.

Option C (Pyrazinamide) is incorrect. Pyrazinamide can cause gout as a result of elevated uric acid levels. Malaise, fever, and gastrointestinal upset are also common.

Option D (Pyridoxine) is incorrect. This is vitamin B6 and is administered to reduce the risk of peripheral neuropathy. Isoniazid binds to pyridoxine and thus, supplementation is required.

Option E (Rifampin) is incorrect. Rifampin classically causes bodily fluids to turn orange. Rarely it can cause liver toxicity.

High-yield Hit 1
ETHAMBUTOL
Ethambutol has no effect on organisms other than mycobacteria. It is taken up by the bacteria and after a period of 24 hours it inhibits their growth. The mechanism of action is unknown. Resistance emerges rapidly if the drug is used on its own.
It is given orally and is well absorbed, reaching therapeutic plasma concentrations within 4 hours. In the blood it is taken up by erythrocytes and slowly released. It is partly metabolised and is excreted in the urine. The half-life is 3-4 hours. It can reach therapeutic concentrations in the CSF in tuberculous meningitis.
Unwanted effects
Unwanted effects are uncommon, the most important being optic neuritis, which is dose related and is more likely to occur if renal function is decreased. It results in visual disturbances: initially red/green colour blindness followed by a decrease in visual acuity. Colour vision should be monitored during prolonged treatment.

From Pharmacology 5E by Rang et al
High-yield Hit 2
PYRAZINAMIDE
Pyrazinamide is inactive at neutral pH but tuberculostatic at acid pH. It is effective against the intracellular organisms in macrophages, since, after phagocytosis, the organisms are contained in phagolysosomes in which the pH is low. Resistance develops rather readily but cross-resistance with isoniazid does not occur.
The drug is well absorbed after oral administration and is widely distributed, penetrating well into the meninges. It is excreted through the kidney, mainly by glomerular filtration.
Unwanted effects
Unwanted effects include gout, which is associated with high concentrations of plasma urates. Gastrointestinal upsets, malaise and fever are reported. With the high doses previously used, serious hepatic damage was a possibility; this is now less likely with lower doses and shorter courses, but nevertheless, liver function should be assessed before treatment.

From Pharmacology 5E by Rang et al
High-yield Hit 3
Antituberculosis drugs

* To avoid emergence of resistant organisms, compound therapy is used, e.g. three drugs initially, then two drugs later.

First-line drugs

* Isoniazid kills actively growing mycobacteria within host cells; mechanism of action unknown. Given orally it penetrates necrotic lesions, also the cebrospinal fluid (CSF). 'Slow acetylators' (genetically determined) respond well. It has low toxicity. Pyridoxine deficiency increases risk of neurotoxicity. No cross-resistance with other agents.
* Rifampicin is a potent, orally active drug that inhibits mycobacterial RNA polymerase. It penetrates CSF. Unwanted effects are infrequent (but serious liver damage has occurred). It induces hepatic drug-metabolising enzymes. Resistance can develop rapidly.
* Ethambutol inhibits growth of mycobacteria by unknown mechanism. It is given orally and can penetrate CSF. Unwanted effects are uncommon; optic neuritis can occur. Resistance can emerge rapidly.
* Pyrazinamide is tuberculostatic against intracellular mycobacteria by an unknown mechanism. Given orally, it penetrates CSF. Resistance can develop rapidly. Unwanted effects: increased plasma urate, liver toxicity with high doses.

Second-line drugs

* Capreomycin is given intramuscularly. Unwanted effects include damage to kidney and to VIIIth nerve.
* Cycloserine is broad spectrum. It inhibits an early stage of peptidoglycan synthesis. Given orally it penetrates the CSF. Unwanted effects affect mostly CNS.
* Streptomycin, an aminoglycoside antibiotic, acts by inhibiting bacterial protein synthesis. It is given intramuscularly. Unwanted effects are ototoxicity (mainly vestibular) and nephrotoxicity. Infrequently used now.


From Pharmacology 5E by Rang et al
A 60-year-old nulliparous woman is seen by her gynecologist for a followup appointment to discuss the results of an endometrial biopsy performed after a routine Papanicolaou (PAP) smear showed high-grade atypia. Pathology reports the endometrial sample is positive for carcinoma. What is the single most important factor in establishing the patient's prognosis?

Answer Choices Correct AnswerCorrect answer Your AnswerYour answer
A. Depth of invasion
B. Histologic grade
C. Pelvic node metastasis
D. Positive peritoneal washings
E. Tumor volume
Option B (Histologic grade) is correct. There is an increased likelihood of extrauterine tumor spread with poorly differentiated tumor and thus substantially worsened prognosis.

Option A (Depth of invasion) is incorrect. Depth of invasion is the second most important prognostic factor. If the carcinoma has invaded more than one third of the myometrium, the prognosis becomes significantly worsened.

Option C (Pelvic node metastasis) is incorrect. Positive peritoneal washings, tumor volume, and pelvic node metastasis are all factors in prognosis, but are not as important as histologic grade and depth of invasion in determining survival rates.

Option D (Positive peritoneal washings) is incorrect. Positive peritoneal washings, tumor volume, and pelvic node metastasis are all factors in prognosis, but are not as important as histologic grade and depth of invasion in determining survival rates.

Option E (Tumor volume) is incorrect. Positive peritoneal washings, tumor volume, and pelvic node metastasis are all factors in prognosis, but are not as important as histologic grade and depth of invasion in determining survival rates.

High-yield Hit 1
Endometrial carcinoma
Figure 12.19 Staging and prognosis of endometrial carcinoma.
The staging of endometrial carcinoma is shown in Fig. 12.19. Early-stage carcinoma can be cured by hysterectomy and bilateral salpingo-oophorectomy (removal of the uterus, fallopian tubes, and both ovaries) followed by radiotherapy. Patients with inoperable carcinoma may benefit from high doses of progesterone and/or radiotherapy.
Other uterine carcinomas
Rarely, malignancy may develop from the stroma of the endometrium, usually with a poor prognosis. They present in a similar manner to endometrial carcinoma but an epithelial component is often present. There are three main types:

* Endometrial stromal sarcoma-consisting of stromal spindle cells.
* Adenosarcoma-malignant stromal and benign epithelial components.
* Carcinosarcoma-malignant stromal and epithelial components; may contain nonuterine tissues.

Leiomyosarcomas are extremely rare, malignant tumors of the myometrium that tend to occur after menopause. They often metastasize by vascular spread to the lungs.

From Crash Course: Endocrine & Reproductive Systems by Meszaros et al
High-yield Hit 2
16. What are the prognostic indicators in endometrial cancer?
Stage of disease at diagnosis is the most significant prognostic factor in patients with endometrial cancer. Younger women have a better prognosis than older women. Some histologic subtypes also confer a worse prognosis, such as papillary serous and clear cell. Histologic grade and myometrial depth of invasion correlate strongly with prognosis. High-grade tumors and deep invasion are associated with increased risk for lymph node metastasis, positive washings, and local recurrence. Other factors, such as the presence of lymph vascular space invasion and the lack of estrogen or progesterone receptor expression, have been shown to confer a worse prognosis. Tumors larger than 2 cm have a greater risk of lymph node metastasis.
17. What is the survival for patients with endometrial cancer?
The overall 5-year survival for patients with endometrial cancer is about 60-70%. However, patients with early-stage disease fare much better than those with late-stage disease (see Table 28-3).

From Ob/Gyn Secrets 3E by Bader
High-yield Hit 3
Table 42-2. 1988 FIGO staging of endometrial carcinoma
Stage
Stage Ia Tumor limited to endometrium
Stage Ib Invasion through less than one half of the myometrium
Stage Ic Invasion equal to or more than half of the myometrium
Stage IIa Endocervical glandular involvement only
Stage IIb Cervical stroma invasion
Stage IIIa Tumor invades serosa or adnexa, or both, or positive peritoneal cytologic findings, or both
Stage IIIb Vaginal metastases
Stage IIIc Metastases to pelvic or para-aortic lymph nodes, or both
Stage IVa Tumor invasion of bladder or bowel mucosa, or both
Stage IVb Distant metastases including intraabdominal or inguinal lymph nodes, or both
Histologic grade does not change the stage
Grade 1 Well differentiated
Grade 2 Moderately differentiated
Grade 3 Poorly differentiated

From Essentials of Obstetrics & Gynecology 4E by Hacker et al
An 83-year-old woman is brought to the emergency department by her daughter because of a large bruise on her arm. The bruise has developed in the past 2 hours, after the patient hit her arm against the corner of a kitchen counter. The patient’s medical history is notable for mild congestive heart failure and chronic atrial fibrillation. The patient is taking the following medications: warfarin, furosemide, atenolol, and ibuprofen. Vital signs are temperature 39.2°C (102.5°F), blood pressure 95/70 mm Hg, heart rate 110 beats/minute, and respiratory rate 17 breaths/minute. There is a large ecchymosis on the left lower arm. An electrocardiogram shows sinus tachycardia. A complete blood count shows a white blood cell count of 19,000/mm3 with 40% bands. A coagulation panel shows platelets 34,000/mm3, prothrombin time 19 seconds, partial thromboplastin time 77 seconds, and bleeding time 16 minutes. Which of the following is the most likely underlying cause of the patient’s acute condition?

Answer Choices Correct AnswerCorrect answer Your AnswerYour answer
A. Atenolol idiosyncratic reaction
B. Ibuprofen overdose
C. Infection
D. Myelodysplastic syndrome
E. Warfarin overdose
Option C (Infection) is correct. This picture is classic for disseminated intravascular coagulation, a condition in which fibrinolysis and coagulation are stimulated, causing pathologic thrombosis and bleeding sequelae. In this elderly patient with fever and bandemia, one must be concerned about an occult infection. Blood cultures, urine cultures, and chest radiography would constitute a crucial part of the workup.

Option A (Atenolol idiosyncratic reaction) is incorrect. The clinical evidence indicates disseminated intravascular coagulation (DIC), most likely from an infection. Atenolol is not noted to cause DIC or other coagulopathies.

Option B (Ibuprofen overdose) is incorrect. Acetylsalicylic acid is the nonsteroidal anti-inflammatory drug (NSAID) associated most strongly with coagulopathy. NSAIDs would not cause prothrombin time and partial thromboplastin time prolongation and would not cause thrombocytopenia.

Option D (Myelodysplastic syndrome) is incorrect. This is a group of disorders in which hematopoietic cells are unable to differentiate normally, leading to various cytopenias. Affected people are elderly, and they usually present with a macrocytic anemia and symptoms of fatigue. This patient’s presentation is much too acute for this to be myelodysplastic syndrome alone.

Option E (Warfarin overdose) is incorrect. Warfarin prolongs the prothrombin time, but the partial thromboplastin time and the bleeding time should be unaffected.

High-yield Hit 1
DISSEMINATED INTRAVASCULAR COAGULATION
Table 52-3. Causes of Disseminated Intravascular Coagulation
Sepsis or Endotoxin
Gram-negative bacteremia
Tissue Damage
Trauma
Closed head injury
Burns
Hypoperfusion or hypotension
Malignant Disease
Adenocarcinoma
Acute promyelocytic leukemia
Primary Vascular Disorders
Vasculitis
Giant hemangioma (Kasabach-Merritt)
Aortic aneurysm
Cardiac mural thrombus
Exogenous Causes
Snake venom
Activated factor infusions (prothrombin complex concentrate)
One of the most common and potentially life-threatening causes of nonimmune peripheral platelet destruction is DIC, which is associated with sepsis, malignancy, advanced liver disease, and other disorders that trigger endotoxin release or cause severe tissue damage (Table 52-3). In DIC caused by bacterial sepsis, circulating endotoxin induces expression of tissue factor on circulating monocytes and endothelial cells, a process leading to overwhelming thrombin and fibrin generation. Deposition of fibrin occurs throughout the vasculature, with relatively inadequate concurrent fibrinolysis, and leads to a thrombotic or microangiopathic vasculopathy and subsequent organ damage. Thrombin activation of platelets and circulating factors eventually overwhelms the bone marrow and liver synthetic capability, respectively, and results in thrombocytopenia and prolongation of the PT and PTT. Thus, although the primary lesion of DIC is clot generation, the clinical end point is usually consumptive coagulopathy. The thrombocytopenia and low factor levels resulting from the consumptive coagulopathy cause mucosal bleeding, especially in the GI tract, and characteristic oozing from intravenous puncture sites.
In the consumptive coagulopathy of DIC, fibrinogen levels are usually low, but they may be normal or slightly high; the acute phase reaction to sepsis or the underlying disorder may actually increase fibrinogen secretion and may lead to normal levels in the midst of DIC. Therefore, DIC should not be ruled out because fibrinogen is in the normal range. Fibrinolysis in DIC is triggered by fibrin clot and tissue-type plasminogen activator; laboratory testing usually shows increased levels of fibrin split products to more than 40 μg/mL (cleavage of fibrin monomers) and D-dimer to more than 0.5 μg/mL (cleavage of fibrin-fibrin bonds). Although fibrin split products are usually elevated in DIC, this finding is nonspecific; in contrast, an elevated D-dimer is more specific for DIC and is often used to confirm the elevated fibrin split product screening assay.
Chronic DIC may be triggered by consumption of platelets and factors into large clots found in aneurysms, hemangiomas, and mural thrombi. A unique cause of chronic DIC is malignant disease, often adenocarcinoma or acute promyelocytic leukemia; malignant cells in these disorders secrete substances that either activate factor X or simulate factor Xa activity. Xa activation leads to formation of the prothrombinase complex, production of thrombin, and platelet activation and clearance; chronic DIC in this circumstance usually causes enough factor consumption that both the PT and the PTT are slightly prolonged. Clinically, such patients present with migratory thrombophlebitis (Trousseau's syndrome) or nonbacterial thrombotic (marantic) endocarditis.
Therapy of DIC should be aimed at the following: (1) treatment of the underlying disorder, such as antibiotics for sepsis or chemotherapy for malignant disease; (2) supportive hemostatic therapy, including platelets, cryoprecipitate (for fibrinogen), and fresh-frozen plasma; and (3) disrupting activation of coagulation factors and platelets. For the last approach, anticoagulation is generally not indicated unless the balance of procoagulant versus anticoagulant activity actively favors clotting, such as arterial thromboemboli with mural thrombus or migratory thrombophlebitis with Trousseau's syndrome. These thrombotic complications of chronic DIC are often resistant to warfarin therapy; resolution of DIC generally requires more intensive anti-Xa therapy (UFH or LMWH), as well as successful treatment of the underlying malignant disease or consumptive disorder. Besides anti-Xa agents, experimental therapies for sepsis and DIC show promise; activated protein C was shown to decrease mortality associated with sepsis/DIC; studies of activated protein C in other pathophysiologies are ongoing.

Taken from Cecil Essentials of Medicine 6E by Andreoli et al
A 28-year-old woman is in the emergency department with a high fever. She felt well until 2 hours ago when she abruptly “began feeling terrible” with shaking chills, headache, muscle aches, nausea, vomiting, and diarrhea. This is the sickest she has ever felt in her life. She has no previous health problems, is on no medications, and does not take birth control pills. She has a temperature of 39.3°C (102.8°F), blood pressure (BP) of 90/60 mm Hg, and heart rate (HR) of 125. Her palms and soles are erythematous. Labs show a leukocytosis with a left shift, increased blood urea nitrogen (BUN), and increased creatinine. Her urinalysis is positive for microscopic hematuria and myoglobinuria. What is this patient’s most likely diagnosis?

Answer Choices Correct AnswerCorrect answer Your AnswerYour answer
A. Cellulitis
B. Erysipelas
C. Neuroleptic malignant syndrome (NMS)
D. Stevens-Johnson syndrome
E. Toxic shock syndrome (TSS)
Option E (Toxic shock syndrome [TSS]) is correct. This patient presents with the classic physical exam findings of TSS. Although other infectious causes of sepsis are possible and should be included in the differential diagnosis and ruled out, TSS is most likely. Although this is not a bacterial skin infection, there is a close relationship between TSS and infection with Staphylococcus aureus. Prolonged tampon use may support bacterial growth and precipitate the syndrome and skin eruption. The patient is not on birth control pills, so it is likely she is using a tampon, which predisposed to this condition. Treatment consists of hospitalization, volume expanders, and appropriate antimicrobial drugs. If organ failure occurs in three or more systems, with the skin signs and no evidence for other causes of sepsis, the syndrome is established.

Option A (Cellulitis) is incorrect. Cellulitis closely resembles erysipelas; it is possible that it can present with toxic or septic types of symptoms. The distribution or location of skin infection is most likely on the lower extremities and would rarely occur on the palms and soles.

Option B (Erysipelas) is incorrect. Although this type of infection could also present similarly, there are distinct differences to the skin manifestations. Generally, there is skin erythema, with distinct sharp borders, induration, and tenderness, usually occurring on the face, arms, or legs.

Option C (Neuroleptic malignant syndrome [NMS]) is incorrect. Although a patient may not admit to taking neuroleptic medication or divulge the condition for which it was prescribed, few of the signs or symptoms of NMS are present. Nor would this syndrome explain the skin manifestations or increased white blood cell count.

Option D (Stevens-Johnson syndrome) is incorrect. This is a desquamative drug reaction, which does not present or develop into the acute onset of hypotension and renal insufficiency seen in this patient.

High-yield Hit 1
Toxic shock syndrome is caused by toxic shock syndrome toxin-producing Staph. aureus
This systemic infection came to prominence through its association with tampon use by healthy women, but it is not confined to women and can occur as a result of Staph. aureus infection at non-genital sites (e.g. a wound). Toxic shock syndrome (TSS) involves multiple organ systems and is characterized by fever, hypotension and a diffuse macular erythematous rash followed by desquamation of the skin, particularly on the soles and palms (Fig. 26.8). TSS is caused by exotoxins of Staph. aureus, most commonly TSST1, which behaves as a superantigen (stimulating T cell proliferation and cytokine release; see Chapter 16). While the prevalence of TSS in the USA is currently estimated at 6000 cases per year, greater than 90% of adults carry antibodies to TSST1. Treatment of TSS includes steps to open the infected site (e.g. drainage), fluid replacement and antistaphylococcal chemotherapy.

Taken from Medical Microbiology 3E by Mims et al
High-yield Hit 2
The disease is initiated with the localized growth of toxin-producing strains of S. aureus in the vagina or a wound, followed by release of the toxin into blood. Toxin production requires an aerobic atmosphere and neutral pH. Clinical manifestations start abruptly and include fever, hypotension, and a diffuse, macular erythematous rash. Multiple organ systems (e.g., central nervous, gastrointestinal, hematologic, hepatic, musculature, renal) are also involved, and the entire skin, including the palms and soles, desquamates (Figure 22-6). As the etiology and epidemiology of this disease have become better understood, the initially high-fatality rate has been decreased to approximately 5%. Unless the patient is specifically treated with an effective antibiotic, however, the risk of recurrent disease is as high as 65%. Serologic studies have demonstrated that more than 90% of adults have antibodies to TSST-1; however, more than 50% of patients with TSS fail to develop protective antibodies after their disease resolves. These unprotected patients are at significant risk for recurrent disease.

Taken from Medical Microbiology 5E by Murray et al
63-year-old woman presents with a 2-week history of malaise, fatigue, fever, and rash. Physical examination reveals a temperature of 38.4°C (101.1°F), grouped petechiae over the distal extremities, and cardiac murmur. Transesophageal echocardiography reveals valvular vegetations and blood cultures grow Streptococcus viridans. What is the most likely predisposing factor for this patient's condition?

Answer Choices Correct AnswerCorrect answer Your AnswerYour answer
A. Bicuspid aortic valve
B. Myxomatous valvular degeneration
C. Postinflammatory valvular disease
D. Prosthetic valves
E. Ventricular septal defect (VSD)
Option B (Myxomatous valvular degeneration) is correct. This patient has infective endocarditis (IE). Myxomatous valvular degeneration describes mitral valve prolapse, which, by virtue of its common presence in the community, is now the overall most common predisposing cardiac factor for IE.

Option A (Bicuspid aortic valve) is incorrect. Congenital bicuspid aortic valves do increase the risk for IE, but it is not the most common predisposing cardiac factor.

Option C (Postinflammatory valvular disease) is incorrect. This describes rheumatic heart disease, which was formally the most common predisposing factor. Therapy with antibiotics has reduced the incidence of rheumatic fever causing it to become a lesser factor in development of IE.

Option D (Prosthetic valves) is incorrect. Individuals with prosthetic valves are at much greater risk for IE. However, as very few individuals have prosthetic valves, it is not the most common predisposing factor.

Option E (Ventricular septal defect [VSD]) is incorrect. A VSD does increase the risk for developing IE, but it is a lesser factor than mitral valve prolapse.

High-yield Hit 1
DIAGNOSTIC CRITERIA FOR INFECTIVE ENDOCARDITIS
The modified Duke criteria for the diagnosis of infective endocarditis are described in Table 8-7.
Table 8-7. Modified Duke Criteria for the Diagnosis of Infective Endocarditis* Comments
Criteria Comments
Major criteria
Microbiologic
Typical microorganism isolated from two separate blood cultures: viridans streptococci, Streptococcus bovis, HACEK group, Staphylococcus aureus, or community-acquired enterococcal bacteremia without a primary focus
or In patients with possible infective endocarditis, at least two sets of cultures of blood collected by separate venipunctures should be obtained within the first 1 to 2 hours of presentation. Patients with cardiovascular collapse should have three cultures of blood obtained at 5- to 10-minute intervals and thereafter receive empirical antibiotic therapy.
Microorganism consistent with infective endocarditis isolated from persistently positive blood cultures
or
Single positive blood culture for Coxiella burnetii or phase I IgG antibody titer to C. burnetii >1:800 C. burnetii is not readily cultivated in most clinical microbiology laboratories.
Evidence of endocardial involvement
New valvular regurgitation (increase or change in preexisting murmur not sufficient)
or
Positive echocardiogram (transesophageal echocardiogram recommended in patients who have a prosthetic valve, who are rated as having at least possible infective endocarditis by clinical criteria, or who have complicated infective endocarditis) Three echocardiographic findings qualify as major criteria: a discrete, echogenic, oscillating intracardiac mass located at a site of endocardial injury; a periannular abscess; and a new dehiscence of a prosthetic valve.
Predisposition to infective endocarditis that includes certain cardiac conditions and injection-drug use Cardiac abnormalities that are associated with infective endocarditis are classified into three groups:
High-risk conditions: previous infective endocarditis, aortic-valve disease, rheumatic heart disease, prosthetic heart valve, coarctation of the aorta, and complex cyanotic congenital heart diseases
Moderate-risk conditions: mitral-valve prolapse with valvular regurgitation or leaflet thickening, isolated mitral stenosis, tricuspid-valve disease, pulmonary stenosis, and hypertrophic cardiomyopathy
Low- or no-risk conditions: secundum atrial septal defect, ischemic heart disease, previous coronary-artery bypass graft surgery, and mitral-valve prolapse with thin leaflets in the absence of regurgitation
Fever Temperature >38° C (100.4° F)
Vascular phenomena Petechiae and splinter hemorrhages are excluded. None of the peripheral lesions are pathognomonic for infective endocarditis.
Immunologic phenomena Presence of rheumatoid factor, glomerulonephritis, Osler's nodes, or Roth's spots
Microbiologic findings Positive blood cultures that do not meet the major criteria
Serologic evidence of active infection; single isolates of coagulase-negative staphylococci and organisms that very rarely cause infective endocarditis are excluded from this category.

*Criteria are adapted from Li JS, Sexton DJ, Mick N, et al: Proposed modifications to the Duke criteria for the diagnosis of infective endocarditis. Clin Infect Dis 30:633-638, 2000.
Cases are defined clinically as definite if they fulfill two major criteria, one major criterion plus three minor criteria, or five minor criteria; they are defined as possible if they fulfill one major and one minor criterion, or three minor criteria.
HACEK, Haemophilus species (Haemophilus parainfluenzae, H. aphrophilus, and H. paraphrophilus), Actinobacillus actinomycetemcomitans, Cardiobacterium hominis, Eikenella corrodens, and Kingella kingae.

From Practical Guide to the Care of the Medical Patient 7E by Ferri
A 29-year-old man presents to the emergency department with severe pain in his left knee that is preventing him from walking. His symptoms first began last night with fevers to 39.8°C (103.6°F) and rigors. Today, he noticed a swollen, erythematous left knee that he was unable to walk on. He admits to intermittent intravenous (IV) drug use. Physical examination reveals an erythematous, hot left knee with significant effusion. There is limitation of passive and active range of motion. What is the most appropriate next step in diagnosis?

Answer Choices Correct AnswerCorrect answer Your AnswerYour answer
A. Blood culture
B. Erythrocyte sedimentation rate (ESR)
C. Joint aspirate
D. Ultrasound of knee
E. X-ray of knee
Option C (Joint aspirate) is correct. This patient presents with an acutely swollen knee and a history of IV drug use. Septic arthritis must be at the very top of the differential diagnosis. All cases of septic arthritis require a joint aspirate for culture and Gram stain, as well as for therapeutic considerations, because removal of some of the fluid decreases the pain.

Option A (Blood culture) is incorrect. Blood culture is important and can yield the etiologic cause of the septic arthritis in the majority of cases. However, joint aspirate should be performed prior.

Option B (Erythrocyte sedimentation rate [ESR]) is incorrect. ESR is elevated in approximately 80% of cases with septic arthritis. However, it is also elevated in cases of osteomyelitis. Although this may be a useful clinical marker, it is not diagnostic and not therapeutic.

Option D (Ultrasound of knee) is incorrect. Ultrasound is useful in hip septic arthritis, particularly in children. However, it is not specific for septic arthritis.

Option E (X-ray of knee) is incorrect. X-ray is not diagnostic in cases of acute septic arthritis, as the duration of symptoms typically does not allow for bony abnormalities to occur.

High-yield Hit 1
Joint aspiration should be carried out in any joint in which infection is suspected. The synovial fluid should be cultured and a quantitative WBC count obtained. A complete blood count with differential and an erythrocyte sedimentation rate also should be obtained. A leukocyte count exceeding 50,000/μL with more than 80% polymorphonuclear leukocytes strongly supports the diagnosis of bacterial arthritis. A WBC count exceeding 100,000/μL in synovial fluid is considered diagnostic of septic arthritis. In patients with septic arthritis, synovial fluid viscosity is often diminished, and a poor mucin clot is found (see Table 42-2). Cultures are positive in nearly 100% of patients with nongonococcal bacterial arthritis but in only about 25% to 50% of those with gonococcal infections.

From Essentials of Surgery by Becker & Stucchi
A 26-year-old man visits his community health clinic complaining of loss of appetite, nausea, vomiting, muscle aches, and joint pain. These symptoms began 3 days ago. He also complains of itchy skin and an aversion to cigarette smoke, which is unusual considering his 10 pack per year smoking history. There is no contributory medical history. The man is a tree trimmer who has returned from a 1-month stint of debris removal from a hurricane disaster zone. Vital signs are temperature: 38.3°C (101.0°F), blood pressure (BP): 125/80, heart rate (HR): 70 beats/min, and respiratory rate (RR): 14 breaths/min. Physical exam is notable for scleral icterus and hepatomegaly. What is the strongest indication for serological testing of individuals with whom the patient had close contact during his stay in the disaster zone?

Answer Choices Correct AnswerCorrect answer Your AnswerYour answer
A. Alanine aminotransferase (ALT) and aspartate aminotransferase (AST) levels greater than 10,000 mIU/mL
B. Complete blood count (CBC) shows 50% lymphocytes
C. Elevated prothrombin time
D. Pancytopenia
E. Positive anti-HAV (hepatitis A virus) IgM antibody test
Option E (Positive anti-HAV (hepatitis A virus) IgM antibody test) is correct. Hepatitis A is the only viral hepatitis that is commonly contracted through contaminated food and water in unsanitary conditions. Persons in close quarters with the patient in a disaster zone are likely to have had similar exposure to the virus. Positive anti-HAV IgM antibody test is sensitive and specific for hepatitis A infection.

Option A (Alanine aminotransferase (ALT) and aspartate aminotransferase (AST) levels greater than 10,000 mIU/mL) is incorrect. Levels of ALT and AST greater than 10,000 mIU/mL are not unusual in viral hepatitis patients. The levels are not specific for hepatitis A.

Option B (Complete blood count (CBC) shows 50% lymphocytes) is incorrect. Mild lymphocytosis is frequently seen in acute hepatitis and is not specific for hepatitis A.

Option C (Elevated prothrombin time) is incorrect. Elevated prothrombin time in suspected hepatitis is an indication of severe disease with significant impairment of hepatic function. It is rare in Hepatitis A patients.

Option D (Pancytopenia) is incorrect. Pancytopenia occurs rarely in hepatitis A infection. This finding is nonspecific for viral causes of hepatitis

High-yield Hit 1
10. How is hepatitis A diagnosed?
The diagnosis of hepatitis A depends on identifying a specific immunoglobulin M (IgM) antibody directed against the viral capsid protein. This is often identified as a hepatitis A viral antibody-IgM (HAVAB-M) class test on lab order sheets. The IgM antibody appears early in infection and persists for 3-6 months. The other available lab test detects the immunoglobulin G (IgG) form of the antibody, which provides diagnosis of past infection and does not have a role in routine clinical practice. Some laboratories offer a combined (total) IgM + IgG, which complicates interpretation of positive results.

From GI/Liver Secrets 3E by McNally
High-yield Hit 2
KEY POINTS: HEPATITIS A VIRUS

1. Fecal-oral transmission
2. Common in both United States and developing countries
3. Diagnosed with IgM antibody
4. Supportive treatment
5. Self-limited infection


From GI/Liver Secrets 3E by McNally
A 27-year-old man presents to the emergency room with a cough, chest pain, and shortness of breath for the last 2 days. He had been previously well and takes no regular medications. He recently arrived in the United States from the Basque region of Spain where he works on a farm as an animal breeder. On arrival, his blood pressure, is 118/75 mm Hg; pulse, 105 beats/min; temperature, 38.8°C (101.8°F); and respirations, 32 breaths/min. There are diffuse wheezes, rhonchi, and rales bilaterally over the lung fields. A chest X-ray (CXR) reveals diffuse, bilateral patchy consolidation.

Answer Choices Correct AnswerCorrect answer Your AnswerYour answer
A. Chlamydia pneumoniae
B. Chlamydia psittaci
C. Coccidioides immitis
D. Coxiella burnetii
E. Cryptococcus neoformans
F. Francisella tularensis
G. Haemophilus influenzae
H. Histoplasma capsulatum
I. Legionella pneumophila
J. Staphylococcus aureus
K. Streptococcus pneumoniae
Option D (Coxiella burnetii) is correct. The suggestion that this individual works as an animal breeder on a farm is a clue to the possibility of Q fever caused by Coxiella burnetii. The name Q fever comes from the term query and is appropriate because there are many different presentations of Q fever. It may present as a flu-like illness, as a hepatitis or as a pneumonia. Pneumonia is the most common presentation in individuals from the Basque region of Spain and in Nova Scotia. The CXR is nonspecific and the only reliable method for diagnosis is serology.

High-yield Hit 1
Coxiella burnetii is the rickettsial cause of Q fever
The disease Q fever was first recognized in Australia in 1935, but the cause was unknown for several years-hence Q (query) fever. The causative rickettsia, Coxiella burnetii, differs from other rickettsiae (see Chapter 27) in the following ways:

* It is not transmitted to humans by arthropods.
* It is relatively resistant to desiccation, heat and sunlight, and is therefore stable enough to be acquired from infected material by inhalation.
* Its main site of action is the lung rather than vascular endothelium elsewhere in the body, so that there is usually no rash.

C. burnetii is transmitted to man by inhalation
C. burnetii can infect many species of wild and domestic animals. In many countries (e.g. USA) infection of livestock is quite common, but there are few human cases (less than 50/year reported in the USA). People who come into contact with infected animals, especially their placentas (e.g. veterinarians, farmers, abattoir workers) are at risk from aerosolized organisms. Unpasteurized milk and tissue fluids from infected stock can also transmit the disease.
After inhalation, the microbe multiplies in the terminal airways of the lung, and about 3 weeks later the patient develops fever, severe headache, and often respiratory symptoms and an atypical pneumonia. The rickettsia can also spread to the liver, commonly causing hepatitis. Recovery is usually complete in 2 weeks, but the disease can become chronic. The heart is then sometimes involved (endocarditis), with thrombocytopenia and purpura in some patients, and this condition is fatal if untreated.
Q fever is diagnosed serologically and treated with antibiotics
A four-fold or greater rise in complement-fixing antibody titer is significant. There are two antigenic forms of the rickettsial lipopolysaccharide (LPS): phase 1 and phase 2. Antibody to phase 2 is seen in acute Q fever, and to both phase 1 and phase 2 in chronic disease. The Weil-Felix test (see Chapter 27) is not used.
Acute infection is treated with oral tetracyclines, chronic infections may require drug combinations such as rifampin and doxycycline or trimethoprim-sulfamethoxazole. A killed vaccine is available for those at risk. The rickettsiae are destroyed when milk is pasteurized.

From Medical Microbiology 3E by Mims et al
A 29-year-old man who is human immunodeficiency virus (HIV)–positive sees his infectious disease physician for ongoing treatment of the HIV. He is on a recommended antiviral protocol and has suffered no opportunistic infections in the past 6 months. No significant findings are evident on physical examination. He has a CD4 count of 180 cells/μL recorded from blood drawn last week. Treatment with which of the following is recommended at this time?

Answer Choices Correct AnswerCorrect answer Your AnswerYour answer
A. Clarithromycin and fluconazole
B. Pentamidine and fluconazole
C. Pneumococcal and influenza vaccines
D. Pneumococcal vaccine and ganciclovir
E. Trimethoprim/sulfamethoxazole Isoniazid and rifampin
Option C (Pneumococcal and influenza vaccines) is correct. In 1996, the U.S. Public Health Service and the Infectious Diseases Society of America issued guidelines for preventing opportunistic infections in patients with HIV. Decisions about when a patient should receive prophylaxis depend on the CD4 count, the presence of symptoms, and drug side effects, among other factors. Currently, patients with CD4 counts less than 200 cells/μL should be on chemoprophylaxis (trimethoprim/sulfamethoxazole) against Pneumocystis carinii and are strongly encouraged to receive the pneumococcal and influenza vaccines.

Option A (Clarithromycin and fluconazole) is incorrect. Clarithromycin is being used as prophylaxis against Mycobacterium avium complex in patients with CD4 counts less than 75 cells/μL. Fluconazole is generally not indicated, except in some patients with advanced HIV disease.

Option B (Pentamidine and fluconazole) is incorrect. Trimethoprim/sulfamethoxazole should be given as chemoprophylaxis against Pneumocystis carinii. Fluconazole is not routinely indicated for patients at this point, except in some patients with advanced HIV disease.

Option D (Pneumococcal vaccine and ganciclovir) is incorrect. Pneumococcal vaccine could be given to this patient, but ganciclovir is not routinely indicated for patients at this point, except to treat active cytomegalovirus disease, such as retinitis.

Option E (Trimethoprim/sulfamethoxazole, isoniazid, and rifampin) is incorrect. Patients with CD4 counts less than 200 cells/μL should be on chemoprophylaxis (trimethoprim/sulfamethoxazole) against Pneumocystis carinii. Not enough information is given here, but isoniazid should be given if the PPD reaction is greater than or equal to 5 mm or a patient has been exposed to a known case of active tuberculosis (TB). Rifampin is used instead if there is a risk of isoniazid-resistant TB.

High-yield Hit 1
PREVENTIVE REGIMEN
PATHOGEN INDICATION FIRST CHOICE ALTERNATIVE
I. STRONGLY RECOMMENDED AS STANDARD OF CARE
Pneumocystis carinii CD4+counts of <200/μL or oropharyngeal candidiasis Trimethoprim-sulfamethoxazole (TMP-SMZ), 1 double-strength tablet (DS) by mouth, daily or TMP-SMZ, 1 single-strength tablet (SS) by mouth daily Dapsone, 50 mg by mouth, twice daily or 100 mg by mouth daily; dapsone, 50 mg by mouth daily plus pyrimethamine, 50 mg by mouth weekly plus leucovorin, 25 mg by mouth weekly; dapsone, 200 mg by mouth plus pyrimethamine, 75 mg by mouth plus leucovorin, 25 mg by mouth weekly; aerosolized pentamidine, 300 mg monthly via Respirgard II nebulizer (Marquest, Englewood, CO); atovaquone, 1500 mg by mouth daily; TMP-SMZ, 1 DS by mouth three times weekly
Mycobacterium tuberculosis Tuberculin skin test (TST) reaction ≥5 mm or prior positive TST result without treatment or contact with person with active tuberculosis, regardless of TST result Isoniazid, 300 mg by mouth plus pyridoxine, 50 mg by mouth daily for 9 mos or isoniazid, 900 mg by mouth plus pyridoxine, 100 mg by mouth, twice weekly for 9 mos Rifampin, 600 mg by mouth daily for 4 mos or rifabutin 300 mg by mouth daily for 4 mos; pyrazinamide, 15-20 mg/kg body weight by mouth daily for 2 mos plus either rifampin, 600 mg by mouth daily for 2 mos or rifabutin, 300 mg by mouth daily for 2 mos
Isoniazid-sensitive
Isoniazid-resistant Same as previous pathogen; increased probability of exposure to isoniazid-resistant tuberculosis Rifampin, 600 mg by mouth daily or rifabutin, 300 mg by mouth daily for 4 mos Pyrazinamide, 15-20 mg/kg body weight by mouth daily for 2 mos plus either rifampin, 600 mg by mouth daily for 2 mos or rifabutin, 300 mg by mouth daily for 2 mos
Multidrug-resistant (isoniazid and rifampin) Same as previous pathogen; increased probability of exposure to multidrug-resistant tuberculosis Choice of drugs requires consultation with public health authorities; depends on susceptibility of isolate from source patient -
Toxoplasma gondii Immunoglobulin G (IgG) antibody to Toxoplasma and CD4+count of <100/μL TMP-SMZ, 1 DS by mouth daily TMP-SMZ, 1 SS by mouth daily; dapsone, 50 mg by mouth daily plus pyrimethamine, 50 mg by mouth weekly plus leucovorin, 25 mg by mouth weekly; dapsone, 200 mg by mouth plus pyrimethamine, 75 mg by mouth plus leucovorin, 25 mg by mouth weekly; atovaquone, 1500 mg by mouth daily with or without pyrimethamine, 25 mg by mouth daily plus leucovorin, 10 mg by mouth daily
Mycobacterium avium complex CD4+count of <50/μL Azithromycin, 1200 mg by mouth weekly or clarithromycin, 500 mg by mouth twice daily Rifabutin, 300 mg by mouth daily; azithromycin, 1200 mg by mouth daily plus rifabutin, 300 mg by mouth daily
Varicella-zoster virus (VZV) Substantial exposure to chickenpox or shingles for patients who have no history of either condition or, if available, negative antibody to VZV Varicella-zoster immune globulin (VZIG), 5 vials (1.25 mL each) intramuscularly, administered ≤96 hours after exposure, ideally in ≤48 hours -
II. USUALLY RECOMMENDED
Streptococcus pneumoniae CD4+count of ≥200/μL 23-valent polysaccharide vaccine, 0.5 mL intramuscularly -
Hepatitis B virus All susceptible patients (i.e., antihepatitis B core antigen-negative) Hepatitis B vaccine: 3 doses -
Influenza virus All patients (annually, before influenza season) Inactivated trivalent influenza virus vaccine: one annual dose (0.5 mL) intramuscularly Oseltamivir, 75 mg by mouth daily (influenza A or B); rimantadine, 100 mg by mouth twice daily, or amantadine, 100 mg by mouth twice daily (influenza A only)
Hepatitis A virus All susceptible patients at increased risk for hepatitis A infection (i.e., antihepatitis A virus-negative) (e.g., illegal drug users, men who have sex with men, hemophiliacs) or patients with chronic liver disease, including chronic hepatitis B or C Hepatitis A vaccine: two doses -
III. EVIDENCE FOR EFFICACY BUT NOT ROUTINELY INDICATED
Bacteria Neutropenia Granulocyte-colony-stimulating factor, 5-10 μg/kg body weight subcutaneously daily for 2-4 weeks or granulocyte-macrophage colony-stimulating factor, 250 μg/m2 subcutaneously for 2-4 weeks -
Cryptococcus neoformans CD4+count of <50/μL Fluconazole, 100-200 mg by mouth daily Itraconazole capsule, 200 mg by mouth daily
Histoplasma capsulatum CD4+count of <100/μL, endemic geographic area Itraconazole capsule, 200 mg by mouth -
Cytomegalovirus (CMV) CD4+count of <50/μL and CMV antibody positivity Oral ganciclovir, 1 gm by mouth three times daily -

Taken from Practical Guide to the Care of the Medical Patient 6E by Ferri
A 4-year-old female presents to the emergency room with a 3-day history of episodes of a severe cough. These episodes occur suddenly and occur as a long series of coughs that are associated with a very forceful inspiratory gasp. It is noted by the father that during these episodes, her tongue begins to protrude, and her face turns purple. Afterwards, she usually vomits and is notably lethargic. Prior to the onset of coughing, she experienced upper respiratory congestion, rhinorrhea, sneezing, and lacrimation for 10 days. What is the most appropriate next step in management to prevent transmission of this patient's condition?

Answer Choices Correct AnswerCorrect answer Your AnswerYour answer
A. Active immunization using acellular vaccine
B. Erythromycin to all close contacts
C. Isolation in hospital for 21 days
D. Passive immunization using microbe-specific immunoglobulin
E. Suctioning of nose and oropharynx on a preventive schedule
Option B (Erythromycin to all close contacts) is correct. This patient has pertussis, which is presently in the paroxysmal stage. Rather than explicitly stating “inspiratory whoop,” the term forceful inspiratory gasp is used, but denotes the same clinical sign. Prophylaxis is indicated for all close and household contacts, regardless of age or immunization status. Prophylaxis consists of 14 days of erythromycin, although 5 days of azithromycin has been demonstrated to be effective and is associated with fewer side effects.

Option A (Active immunization using acellular vaccine) is incorrect. Postexposure immunization does not prevent infection of pertussis.

Option C (Isolation in hospital for 21 days) is incorrect. Isolation is an important strategy for prevention of pertussis transmission. However, isolation is usually 5 days.

Option D (Passive immunization using microbe-specific immunoglobulin) is incorrect. Postexposure immunization does not prevent infection of pertussis.

Option E (Suctioning of nose and oropharynx on a preventive schedule) is incorrect. There is no indication for suctioning of the nose and oropharynx either in the treatment or prevention of pertussis.

High-yield Hit 1
Whooping cough is managed with supportive care and erythromycin
Supportive care is of prime importance. Infants are at greatest risk of complications, and admission to hospital should be considered for children under 1 year of age. For specific antibacterial treatment to be effective it must penetrate the respiratory mucosa and inhibit or kill the infecting organism. Erythromycin is the drug of choice. Although the treatment is often not begun until the disease is recognized in the paroxysmal phase, it does appear to reduce its severity and duration. It also reduces the risk of organisms in the throat, thereby helping to reduce the infectivity of the patient, and helps to reduce the risk of secondary infections.
Erythromycin prophylaxis of close contacts of active cases is helpful in controlling the spread of infection.

From Medical Microbiology 3E by Mims et al
High-yield Hit 2
15. Name the three stages of pertussis infection (whooping cough).

1. Catarrhal (1-2 weeks): Low-grade fever, upper respiratory infection symptoms
2. Paroxysmal (2-4 weeks): Severe cough occurring in paroxysms, onset of inspiratory "whoop"
3. Convalescent (1-2 weeks): Resolution of symptoms


From Pediatric Secrets 4E by Polin & Ditmar
Name the three stages of pertussis infection (whooping cough).
1. Catarrhal (1-2 weeks): Low-grade fever, upper respiratory infection symptoms
2. Paroxysmal (2-4 weeks): Severe cough occurring in paroxysms, onset of inspiratory "whoop"
3. Convalescent (1-2 weeks): Resolution of symptoms
A 64-year-old man is seen by the physician for his annual health maintenance examination. He has been well since his mitral valve replacement 2 years ago. At present he has no complaints. His family history is notable for paternal colon cancer at the age of 75 years. A fecal occult blood test is performed and is positive for blood. A colonoscopy is scheduled. What is the most appropriate pharmacotherapy to decrease this patient's risk for infective endocarditis (IE) associated with this procedure?

Answer Choices Correct AnswerCorrect answer Your AnswerYour answer
A. Amoxicillin
B. Ampicillin and gentamicin
C. Clindamycin
D. Gentamicin
E. Vancomycin and gentamicin
Option B (Ampicillin and gentamicin) is correct. This patient is at high risk for developing IE. A colonoscopy can result in significant bacteremia and coverage needs to encompass gram-negative bacteria. Consequently, both ampicillin and gentamicin are used for gastrointestinal and genitourinary procedures.

Option A (Amoxicillin) is incorrect. This is appropriate prophylaxis for high-risk individuals undergoing oral, respiratory, or esophageal procedures

Option C (Clindamycin) is incorrect. Clindamycin is used in high-risk individuals undergoing oral, respiratory or esophageal procedures who are allergic to penicillins.

Option D (Gentamicin) is incorrect. Gentamicin is not used alone in the prophylaxis of IE.

Option E (Vancomycin and gentamicin) is incorrect. Vancomycin should be used as a substitute only when the patient is allergic to penicillins.

High-yield Hit 1
DIAGNOSTIC CRITERIA FOR INFECTIVE ENDOCARDITIS
The modified Duke criteria for the diagnosis of infective endocarditis are described in Table 8-7.
Table 8-7. Modified Duke Criteria for the Diagnosis of Infective Endocarditis* Comments
Criteria Comments
Major criteria
Microbiologic
Typical microorganism isolated from two separate blood cultures: viridans streptococci, Streptococcus bovis, HACEK group, Staphylococcus aureus, or community-acquired enterococcal bacteremia without a primary focus
or In patients with possible infective endocarditis, at least two sets of cultures of blood collected by separate venipunctures should be obtained within the first 1 to 2 hours of presentation. Patients with cardiovascular collapse should have three cultures of blood obtained at 5- to 10-minute intervals and thereafter receive empirical antibiotic therapy.
Microorganism consistent with infective endocarditis isolated from persistently positive blood cultures
or
Single positive blood culture for Coxiella burnetii or phase I IgG antibody titer to C. burnetii >1:800 C. burnetii is not readily cultivated in most clinical microbiology laboratories.
Evidence of endocardial involvement
New valvular regurgitation (increase or change in preexisting murmur not sufficient)
or
Positive echocardiogram (transesophageal echocardiogram recommended in patients who have a prosthetic valve, who are rated as having at least possible infective endocarditis by clinical criteria, or who have complicated infective endocarditis) Three echocardiographic findings qualify as major criteria: a discrete, echogenic, oscillating intracardiac mass located at a site of endocardial injury; a periannular abscess; and a new dehiscence of a prosthetic valve.
Predisposition to infective endocarditis that includes certain cardiac conditions and injection-drug use Cardiac abnormalities that are associated with infective endocarditis are classified into three groups:
High-risk conditions: previous infective endocarditis, aortic-valve disease, rheumatic heart disease, prosthetic heart valve, coarctation of the aorta, and complex cyanotic congenital heart diseases
Moderate-risk conditions: mitral-valve prolapse with valvular regurgitation or leaflet thickening, isolated mitral stenosis, tricuspid-valve disease, pulmonary stenosis, and hypertrophic cardiomyopathy
Low- or no-risk conditions: secundum atrial septal defect, ischemic heart disease, previous coronary-artery bypass graft surgery, and mitral-valve prolapse with thin leaflets in the absence of regurgitation
Fever Temperature >38° C (100.4° F)
Vascular phenomena Petechiae and splinter hemorrhages are excluded. None of the peripheral lesions are pathognomonic for infective endocarditis.
Immunologic phenomena Presence of rheumatoid factor, glomerulonephritis, Osler's nodes, or Roth's spots
Microbiologic findings Positive blood cultures that do not meet the major criteria
Serologic evidence of active infection; single isolates of coagulase-negative staphylococci and organisms that very rarely cause infective endocarditis are excluded from this category.

*Criteria are adapted from Li JS, Sexton DJ, Mick N, et al: Proposed modifications to the Duke criteria for the diagnosis of infective endocarditis. Clin Infect Dis 30:633-638, 2000.
Cases are defined clinically as definite if they fulfill two major criteria, one major criterion plus three minor criteria, or five minor criteria; they are defined as possible if they fulfill one major and one minor criterion, or three minor criteria.
HACEK, Haemophilus species (Haemophilus parainfluenzae, H. aphrophilus, and H. paraphrophilus), Actinobacillus actinomycetemcomitans, Cardiobacterium hominis, Eikenella corrodens, and Kingella kingae.
SPECIAL DIAGNOSTIC CONSIDERATIONS

1. Right-sided endocarditis
1. Usually seen in intravenous drug users.
2. Physical examination: may reveal a murmur of tricuspid regurgitation (holosystolic, heard at left lower sternal border, increased by inspiration, decreased by expiration and Valsalva's maneuver); evidence of failure of right side of heart may also be present (neck vein distention, congestive hepatomegaly, edema).
3. Blood cultures: S. aureus (50% of cases), streptococci and enterococci (20% of all cases), and gram-negative bacilli (approximately 10% of cases)
4. Chest x-ray: may demonstrate peripheral wedge-shaped infiltrates with cavitation (septic pulmonary emboli).
5. Right-sided endocarditis is less aggressive than left-sided disease; the prognosis for patients with isolated tricuspid valve endocarditis caused by S. aureus is generally favorable, with good response to medical therapy. A penicillase-resistant penicillin (e.g., cloxacillin 2 g IV q4h) is effective for most patients with isolated tricuspid endocarditis caused by methicillin-susceptible S. aureus. Ceftriaxone is frequently used for outpatient intravenous therapy (if S. aureus is sensitive), since it can be given once daily.
2. Prosthetic valve endocarditis
1. Overall frequency is approximately 2%.
2. Overall mortality rate is 59%.
3. The microbiologic agent involved and the mortality rate are related to the time of onset of endocarditis after cardiac valve implantation.
1. Early-onset prosthetic valve endocarditis (within 2 months of implantation)
* (a) Frequency of endocarditis is 0.78%.
* (b) Usually resulting from surgical infection
* (c) Staphylococci are the most common organisms (S. epidermidis the predominant one); they are usually resistant to cephalosporins and semisynthetic penicillinase-resistant penicillins.
* (d) Mortality rate is 77%.
* (e) Replacement of the valve is usually necessary.
2. Late-onset prosthetic valve endocarditis (occurring more than 2 months postoperatively)
* (a) Frequency of endocarditis is 1.1%.
* (b) Usually community-acquired infection
* (c) Streptococci are the predominant organisms.
* (d) Overall mortality rate is 46%.
3. Streptococcus bovis endocarditis
1. Often associated with large bowel lesion, frequently carcinoma
2. A gastrointestinal work-up should be undertaken.

MAJOR COMPLICATIONS OF INFECTIVE ENDOCARDITIS

1. Congestive heart failure caused by valvular destruction or associated myocarditis
2. Embolism
1. Central nervous system (CNS): hemiplegia, sensory loss, aphasia, meningeal irritation, mycotic aneurysm, brain abscesses, seizures, headaches, cerebral hemorrhage (from rupture mycotic aneurysm or stroke)
2. Kidneys: hematuria resulting from focal glomerulonephritis, renal failure resulting from diffuse proliferative glomerulonephritis, renal emboli, and infarction
3. Coronary arteries: heart failure, angina, myocardial infarction
4. Spleen: splenic infarct
5. Musculoskeletal: osteomyelitis and infectious arthritis also possible embolic phenomena
6. Anticoagulation therapy has not been shown to prevent embolization in patients with infective endocarditis. It increases the risk of intracerebral hemorrhage and is contraindicated.
3. Arrhythmias, various degrees of heart block
4. Pericarditis, myocardial abscess (perivalvular abscess), myocarditis

MANAGEMENT

1. Medical:
1. Box 8-2 describes antibiotic treatment of adults, based on positive blood culture results.
1. Antibiotic therapy (after identification of the organism) should be guided by susceptibility testing (minimum inhibitory concentration, minimum bactericidal concentration).
2. Peak serum bactericidal titers of 1:64 or greater and trough bactericidal titers of 1:32 or greater are recommended.
2. Initial intravenous antibiotic therapy (before culture results) is aimed at the most likely organism.
1. In patients with prosthetic valves or patients with native valves but allergic to penicillin: vancomycin plus rifampin and gentamicin
2. In intravenous drug addicts: penicillinase-resistant penicillin (oxacillin or nafcillin) plus gentamicin
3. In native-valve endocarditis: combination of penicillin and gentamicin; a penicillinase-resistant penicillin or vancomycin should be added if acute bacterial endocarditis is present or if S. aureus is suspected as one of the possible causative organisms; the combination of vancomycin and gentamicin provides broad empiric coverage while awaiting culture results.
2. Surgical: indications for cardiac surgery in patients with active native valve infective endocarditis are listed in Box 8-3. Box 8-4 describes indications for surgery in patients with prosthetic valve endocarditis.

PROPHYLAXIS OF INFECTIVE ENDOCARDITIS

1. Cardiac conditions associated with endocarditis are described in Box 8-5.
2. Prophylactic regimens for various procedures are described in Table 8-8.

BOX 8-2 Antibiotic Treatment of Infective Endocarditis
Streptococci
Viridans streptococci and Streptococcus bovis
Penicillin G susceptible (MIC ≤0.1 μg/mL)
Regimen A: Penicillin G at 12-18 million U/d IV in divided doses q4h for 4 weeks
Regimen B: Penicillin as in regimen A plus gentamicin 1 mg/kg IV q8h both for 2 weeks
Regimen C: Penicillin plus gentamicin for 2 weeks as in regimen B with penicillin continued 2 weeks longer
*Regimen D: Ceftriaxone at 2 g IV or IM daily for 4 weeks
*Regimen E: Vancomycin at 15 mg/kg IV q12h for 4 weeks
Relatively penicillin G resistant (MIC >0.1 μg/mL but <0.5 μg/mL)
Regimen C
*Regimen D or E
Enterococci and viridans streptococci (MIC ≥0.5 μg/mL)
Regimen F: Penicillin G 18-30 million U/d or ampicillin at 12 g/d IV in divided doses q4h, plus gentamicin at 1 mg/kg IV q8h or streptomycin at 7.5 mg/kg IM q12h, both for 4 to 6 weeks
*Regimen G: Vancomycin at 15 mg/kg IV q12h plus gentamicin or streptomycin as in regimen F, both for 4 to 6 weeks
Prosthetic valve (see text)
Staphylococci
Native valve
Methicillin susceptible (Staphylococcus epidermidis, Staphylococcus aureus)
Regimen H: Nafcillin at 2 g IV q4h for 4 to 6 weeks with or without gentamicin 1 mg/kg IV q8h for the first 3 to 5 days
*Regimen I: Cefazolin at 2 g IV q8h for 4 to 6 weeks with or without gentamicin as in regimen H
Regimen J: Vancomycin at 15 mg/kg IV q12h for 4 to 6 weeks
Methicillin resistant
Regimen J
Prosthetic valve
Methicillin susceptible
Regimen H, I, or J: for 6 to 8 weeks with gentamicin for the first 2 weeks and rifampin at 300 mg orally q8h for 6 to 8 weeks
Methicillin resistant
Regimen J: for 6 to 8 weeks with gentamicin for the first 2 weeks and rifampin at 300 mg orally q8h for 6 to 8 weeks

*Regimens for patients allergic to penicillin.
MIC, minimal inhibitory concentration.
From Gorbach SL: Infectious Diseases, 2nd ed. Philadelphia, WB Saunders, 1998.
BOX 8-3 Indications for Cardiac Surgery in Patients with Infective Endocarditis

* Indications*Moderate to severe congestive heart due to valve dysfunction
* Partially dehisced unstable prosthetic valve
* Persistent bacteremia in the face of optimal antimicrobial therapy
* Absence of effective bactericidal therapy
* Fungal endocarditis
* Relapse of PVE after optimal antimicrobial therapy
* Persistent unexplained fever (≥10 days) in patient with culture-negative PVE
* Staphylococcus aureus PVE
* Staphylococcus epidermidis PVE

* Relative Indication†Perivalvular extension of infection (myocardial, septal, or annulus abscess, intracardiac fistula)
* Poorly responsive Staphylococcus aureus endocarditis involving the aortic or mitral valve
* Relapse of native valve IE after optimal antimicrobial therapy
* Large (>10 mm diameter) hypermobile vegetations
* Persistent unexplained fever (≥10 days) in patient with culture-negative native valve IE
* Endocarditis due to highly antibiotic-resistant enterococci or gram-negative bacilli

8.13 Acquired Immunodeficiency Syndrome

From Practical Guide to the Care of the Medical Patient 7E by Ferri
appropriate infective endocarditis prophylaxis for high-risk individuals undergoing oral, respiratory, or esophageal procedures

A. Amoxicillin
B. Ampicillin and gentamicin
C. Clindamycin
D. Gentamicin
E. Vancomycin and gentamicin
Option A (Amoxicillin)
used in high-risk for infective endocarditis individuals undergoing oral, respiratory or esophageal procedures who are allergic to penicillins.

A. Amoxicillin
B. Ampicillin and gentamicin
C. Clindamycin
D. Gentamicin
E. Vancomycin and gentamicin
Clindamycin is used in high-risk individuals undergoing oral, respiratory or esophageal procedures who are allergic to penicillins.
A 64-year-old man is seen by the physician for his annual health maintenance examination. He has been well since his mitral valve replacement 2 years ago. At present he has no complaints. His family history is notable for paternal colon cancer at the age of 75 years. A fecal occult blood test is performed and is positive for blood. A colonoscopy is scheduled. What is the most appropriate pharmacotherapy to decrease this patient's risk for infective endocarditis (IE) associated with this procedure if he is allergic to penicillins?

Answer Choices Correct AnswerCorrect answer Your AnswerYour answer
A. Amoxicillin
B. Ampicillin and gentamicin
C. Clindamycin
D. Gentamicin
E. Vancomycin and gentamicin
Option E (Vancomycin and gentamicin)
Vancomycin should be used as a substitute only when the patient is allergic to penicillins.

otherwise use Ampicillin and gentamicin
A 54-year-old man is admitted after having a seizure. He was at home with his family and suddenly felt a tingling sensation migrating up his left arm and then his face. Shortly afterwards, he lost consciousness and had a convulsion that started in his left leg and subsequently involved his whole body, lasting for approximately 3 minutes. By the time emergency medical services (EMS) arrived, he was awake but very tired and confused. His wife remembers him running a low fever lately, and complaining of a headache the day before. His prior medical history is positive for chronic obstructive pulmonary disease (COPD) and a dental extraction 1 week ago. He is a chronic smoker and drinks 1 beer a week. He is drowsy but follows commands. His vital signs are as follows: temperature (T), 39.2°C (102.5°F); blood pressure (BP), 100/60 mm Hg; heart rate (HR), 120 beats/minute; respiration rate (RR), 22 breaths/minute. The funduscopic examination is normal. His pupils are equally reactive and extraocular movements are intact. He is weaker and hyperreflexic on the left side. His neck is rigid. A diastolic murmur is discerned on auscultation. A computed tomography (CT) of the head with contrast shows a 3 × 4 cm ring-enhancing lesion in the right parietal lobe, with edema around it. What is the most appropriate next step in management of this patient?

Answer Choices Correct AnswerCorrect answer Your AnswerYour answer
A. Ceftriaxone
B. Emergent surgery
C. Lumbar puncture
D. Transesophageal echocardiogram (TEE)
E. Vancomycin
Option A (Ceftriaxone) is correct. In the setting of low-grade fevers, malaise, and headache, a solitary ring-enhancing lesion is a brain abscess until proven otherwise, and empirical treatment with antibiotics should be started. Brain abscesses usually develop from contiguous spreads from the sinuses/neck/teeth, or through a breech caused by trauma or neurosurgical procedures. Hematogenous dissemination can also be a cause. Empirical treatment includes third-generation cephalosporin plus metronidazole.

Option B (Emergent surgery) is incorrect. Neurosurgical consult is the next step, after antibiotics have been started. The options include stereotactic aspiration or excision. Surgery is not performed before antibiotics are started, or if the abscess has not formed a capsule yet.

Option C (Lumbar puncture) is incorrect. Lumbar puncture is contraindicated if there is a suspicion of cerebral abscess, because it can a cause rupture of the abscess and even herniation. Unless there already is seeding of the meninges, the cerebrospinal fluid (CSF) would only show nonspecific findings of lymphocytic pleocytosis and elevated protein.

Option D (Transesophageal echocardiogram (TEE)) is incorrect. The patient has a new heart murmur and had a dental procedure in the recent past. Subacute bacterial endocarditis should be ruled out, and a TEE is the best test to evaluate the valves. A clinical examination should include a funduscopy looking for Roth spots, as well as the examination of the nails (splinter hemorrhages) and extremities (Janeway lesions). Three sets of bacterial cultures need to be sent immediately as well.

Option E (Vancomycin) is incorrect. If Staphylococcus aureus is suspected, nafcillin or vancomycin are added. There is no evidence of trauma or prior surgery that would predispose this patient to a Staphylococcal infection.
A 45-year-old man presents to the emergency room with an acute onset of abdominal pain. He reports no significant medical or surgical comorbidity but happens to mention that two days ago, he sustained a rather hard fall while cleaning his 200-gallon fish tank full of sea anemones. The chair upon which he was standing “just slipped out” from underneath him, and he fell onto the corner of the tank. His primary complaints are pain and nausea. There has been no recent diarrhea or vomiting. Physical examination demonstrates a temperature of 38.1° C (100.5° F); severe midepigastric pain, which radiates to the back; and a papular rash on his right wrist. There is no involuntary guarding or rebound tenderness on examination of the abdomen. Radiography of the chest is negative for rib fracture. Laboratory tests are positive for a blood ETOH of 400, a slightly increased WBC count, and increased serum amylase and lipase activities. Which of the following is the most likely diagnosis?

Answer Choices Correct AnswerCorrect answer Your AnswerYour answer
A. Aortic aneurysm
B. Intestinal obstruction/perforation
C. Medication use
D. Perforated ulcer
E. Toxic exposure
Option E (Toxic exposure) is correct. The anemone sting could cause pancreatitis, and given his physical and laboratory findings along with a rash, this needs to be considered.

Option A (Aortic aneurysm) is incorrect. This must be considered in the differential diagnosis of pain in the back, but there are other things to rule out first.

Option B (Intestinal obstruction/perforation) is incorrect. This is unlikely given the history. The pain is also epigastric, and the abdomen is not acute for perforation as a complication.

Option C (Medication use) is incorrect. Although this is part of the differential diagnosis, it is lower on the list given the other historical clues.

Option D (Perforated ulcer) is incorrect. Pain radiating through to the back could be caused by a perforated ulcer, but it is not an acute abdomen, and this is not supported by the history.

High-yield Hit 1
COELENTERATES
Jellyfish and Anemones. Jellyfish and anemones belong to the Cnidaria phylum, so named because of their venomous organelles, called cnidae. The cnidae found in jellyfish and anemones (called nematocysts and spirocysts, respectively) are located on exposed tentacles. On tactile stimulation, the tentacles send forth a tethered projectile to deliver venom through the dermis. As the victim's surrounding musculature contracts, the venom is disseminated. The toxins contained in the venom from these organisms have not been fully documented. Hemolysins, DNases, and histamine releasers have been identified in some venoms. Several peptide toxins have been characterized from the sea anemone, Anemonia sulcata, which act similarly to α-scorpion toxins by inactivating the sodium channel.
Stings by jellyfish and anemones typically produce immediate pain at the site of envenomation, followed by erythematous and urticarial lesions. Anaphylaxis is not common in most situations unless previous sensitization has occurred. Depending on the severity of the sting, wheals and whiplike patterns at the sites of envenomation may appear within a few minutes or be delayed by several hours, followed in some cases by dermal necrosis. Recurrence of eruptions days after the envenomation has been reported. Systemic reactions may include muscle spasms and cramps, vomiting, nausea, diarrhea, diaphoresis, and in rare cases, cardiorespiratory failure.

Taken from Cecil Textbook of Medicine by Goldman.
High-yield Hit 2
Etiology
In the United States, alcohol consumption is the principal cause of chronic pancreatitis, accounting for approximately 70% of all cases. In contrast to acute pancreatitis, gallstones do not cause chronic pancreatitis. Malnutrition-induced (tropical) pancreatitis is the most prevalent form of chronic pancreatitis in developing Asian and African countries. Consumption of cassava, a plant indigenous to these regions, may contribute to pancreatic injury by increasing serum thiocyanate levels, which subsequently increase cellular free radical production. Ingestion of a diet deficient in micronutrients and antioxidants exposes the pancreas to injury by unopposed free radicals. In a few cases, chronic pancreatitis results from trauma or from prolonged metabolic disturbances, such as hypercalcemia and hypertriglyceridemia. In rare instances, pancreatitis can be inherited as an autosomal dominant disease, presenting as acute or chronic pancreatitis with prominent pancreatolithiasis. Approximately 20% of cases are believed to be idiopathic. A bimodal age distribution in adolescents and the elderly suggests that there may be two distinct pathophysiologic causes. Clinical studies indicate an increased incidence of heterozygotic mutations in the CFTR gene in idiopathic chronic pancreatitis. In contrast to patients with cystic fibrosis, who usually present with painless pancreatic insufficiency, these patients experience recurrent pancreatitis despite normal sweat tests and pulmonary function.

Taken from Cecil Textbook of Medicine by Goldman.
A 73-year-old man is brought in from his nursing home, because he appeared somnolent at breakfast. Physical examination reveals photophobia, nuchal rigidity, positive Kerning sign, and positive Brudzinski sign. What is the most appropriate empiric therapy in this patient?

Answer Choices Correct AnswerCorrect answer Your AnswerYour answer
A. Ampicillin
B. Ceftriaxone
C. Ceftriaxone and ampicillin
D. Ceftriaxone and vancomycin
E. Vancomycin
Option C (Ceftriaxone and ampicillin) is correct. This is an elderly patient presenting with classic signs for meningitis. In the elderly, Streptococcus pneumoniae remains the most common cause of meningitis, but Listeria monocytogenes is the second most common cause. Ceftriaxone does not adequately cover Listeria, thus, ampicillin must be added to any empiric therapy for meningitis in the elderly, as well as in neonates.

Option A (Ampicillin) is incorrect. This would not provide adequate coverage against Neisseria meningitis, a common cause of meningitis.

Option B (Ceftriaxone) is incorrect. This is appropriate empiric therapy for adults with suspected meningitis.

Option D (Ceftriaxone and vancomycin) is incorrect. This would be appropriate empiric therapy in a community with reasonably high rates of cephalosporin-resistant pneumococci.

Option E (Vancomycin) is incorrect. Vancomycin alone is not used as empiric therapy for meningitis.

High-yield Hit 1
Clinical features
The features of meningitis are headache, neck stiffness, and clouding of consciousness. Bacterial meningitis is spread by droplet infection, and the organism lodges and multiplies in the nasopharnynx. It enters the bloodstream, giving rise to a generalized septicemia, pyrexia, malaise, and rigors. Signs of cerebral irritability include photophobia, increased tendon reflexes, vomiting, and convulsions. In patients with Neisseria meningitidis, petechia, palpable purpura, and other skin manifestations may appear.
Kernig's sign (Fig. 34.6) may be positive, i.e., pain on passively extending the knee with the hips fully flexed. A petechial rash may occur, especially with meningococcal meningitis.
In tuberculous meningitis, symptoms may initially be nonspecific with malaise, anorexia, headache, and a variable mild fever. These symptoms may persist for days but gradually an unremitting deterioration occurs. There may be personality changes and intermittent dulling of consciousness before signs of meningism are obvious. The appearance of focal neurologic signs suggests a complication (e.g., venous sinus thrombosis, cerebral edema, or hydrocephalus).
Figure 34.6 (A) Eliciting Kernig's sign. (B) Testing for neck stiffness.

From Crash Course: Internal Medicine by Baliga
High-yield Hit 2
Figure 34.7 Changes in the cerebrospinal fluid in meningitis.
Figure 34.8 Treatment of meningitis.

From Crash Course: Internal Medicine by Baliga
presents with a flu-like illness, petechiae, hepatosplenomegaly, and jaundice.

A. Babesiosis
B. Erythema infectiosum
C. Human granulocytic ehrlichiosis
D. Lyme disease
E. Rocky mountain spotted fever (RMSF)
Option A (Babesiosis)
presents in children as a maculopapular rash on the arms that spreads to the face as a “slapped cheek” appearance.

A. Babesiosis
B. Erythema infectiosum
C. Human granulocytic ehrlichiosis
D. Lyme disease
E. Rocky mountain spotted fever (RMSF)
Option B (Erythema infectiosum). Erythema infectiosum, also known as fifth disease, presents in children as a maculopapular rash on the arms that spreads to the face as a “slapped cheek” appearance.
presents with flu-like constitutional symptoms, followed by abdominal pain, arthralgias, and confusion. An erythematous rash is rare.
A. Babesiosis
B. Erythema infectiosum
C. Human granulocytic ehrlichiosis
D. Lyme disease
E. Rocky mountain spotted fever (RMSF)
Option C (Human granulocytic ehrlichiosis). Ehrlichiosis presents with flu-like constitutional symptoms, followed by abdominal pain, arthralgias, and confusion. An erythematous rash is rare.
begins with a flu-like prodrome of 1 to 3 days, followed by a rash that begins on the wrists and ankles. It then spreads to the palms and soles, followed by the trunk and face. Treatment is with doxycycline.
A. Babesiosis
B. Erythema infectiosum
C. Human granulocytic ehrlichiosis
D. Lyme disease
E. Rocky mountain spotted fever (RMSF)
Option E (Rocky mountain spotted fever [RMSF]) is correct. This patient has RMSF, a tick-borne disease that, despite its name, most commonly occurs in North Carolina, Oklahoma, Tennessee, and South Carolina. The clinical picture of RMSF begins with a flu-like prodrome of 1 to 3 days, followed by a rash that begins on the wrists and ankles. It then spreads to the palms and soles, followed by the trunk and face. Treatment is with doxycycline.
A 26-year-old woman presents to the physician with a 2-day history of altered urination. She states she is urinating much more frequently than usual and has also been urinating three to four times a night. When she urinates, she experiences a burning-type pain. She has also noticed a constant urge to urinate, despite having just micturated. She denies any associated discharge, odors, fever, abdominal pain, nausea, or diarrhea. She was previously well and does not take any regular medications. She has not been sexually active for the last 12 months and specifically denies a history of sexually transmitted diseases (STDs). On examination, she is afebrile, the abdomen is soft and nontender and there is no costovertebral angle tenderness. Urine dipstick is positive for leukocyte esterase activity. What is the most likely cause for these findings?

Answer Choices Correct AnswerCorrect answer Your AnswerYour answer
A. Escherichia coli
B. Escherichia faecalis
C. Proteus mirabilis
D. Staphylococcus aureus
E. Staphylococcus saprophyticus
Option A (Escherichia coli) is correct. This patient has a urinary tract infection (UTI), which are overwhelmingly caused by Escherichia coli. The combination of frequency, burning, and urgency in the absence of discharge or other risk factors for STDs, have been determined by meta-analysis to be sufficient to start empiric antibiotic therapy.

Option B (Escherichia faecalis) is incorrect. Enterococci are common causes, but less common than Escherichia coli.

Option C (Proteus mirabilis) is incorrect. Proteus is a common cause ofUTIs. It produces urease, which hydrolyzes urea to ammonia. The ammonia alkalinizes the urine, resulting in the precipitation of phosphate, magnesium, and carbonate, thus promoting struvite stones.

Option D (Staphylococcus aureus) is incorrect. Staphylococcus aureus rarely causes UTIs and usually is a hospital-acquired infection.

Option E (Staphylococcus saprophyticus) is incorrect. This is the second most common cause of UTI) in the United States, but less common in Europe.

High-yield Hit 1
Figure 5.17 Incidence of community- and hospital-acquired urinary tract infections (UTIs) caused by bacteria.

From Crash Course: Renal & Urinary Systems by Stevenson
It produces urease, which hydrolyzes urea to ammonia. The ammonia alkalinizes the urine, resulting in the precipitation of phosphate, magnesium, and carbonate, thus promoting struvite stones.
A. Escherichia coli
B. Escherichia faecalis
C. Proteus mirabilis
D. Staphylococcus aureus
E. Staphylococcus saprophyticus
Option C (Proteus mirabilis
rarely causes UTIs and usually is a hospital-acquired infection.
A. Escherichia coli
B. Escherichia faecalis
C. Proteus mirabilis
D. Staphylococcus aureus
E. Staphylococcus saprophyticus
Option D (Staphylococcus aureus)
This is the second most common cause of UTI) in the United States, but less common in Europe.
A. Escherichia coli
B. Escherichia faecalis
C. Proteus mirabilis
D. Staphylococcus aureus
E. Staphylococcus saprophyticus
Option E (Staphylococcus saprophyticus)
the leading cause of blindness worldwide
Chlamydia trachomatis --> Different subtypes can also cause neonatal conjunctivitis on days 8 to 10 after birth,
presents with a painless chancre.
A. Chlamydia trachomatis
B. Neisseria gonorrhoeae
C. Treponema pallidum
D. Trichomonas vaginalis
E. Ureaplasma urealyticum
Option C (Treponema pallidum) . This is the cause of syphilis, which presents with a painless chancre. The patient does not present with any evidence of genital ulcers.
Young child with sudden onset of fever, petechiae or purpura (black and blue marks) on the skin, malaise, and stiff neck. The cerebrospinal fluid contains intracellular gram-negative diplococci.
Meningococcal meningitis
Sexually active man with greenish-yellow discharge from the urethra has difficulty urinating. Discharge contains gram-negative intracellular diplococci.
Gonorrhea (local infection)
Sexually active man with painful, inflamed joint. Aspirate may contain gram-negative intracellular diplococci.
Disseminated gonococcal infection
Sexually active woman develops fever, abdominal pain, cervical tenderness, and vaginal discharge following unusually severe and prolonged menstrual period. An IUD had been inserted into the patient several years ago.
PID
A 32-year-old woman has developed agranulocytosis as a side effect of the clozapine prescribed to treat her schizophrenia. Today she is admitted to the hospital with a temperature of 39.7°C (103.4°F). The patient’s blood pressure is 80/60 mm Hg, her heart rate is 125 beats/minute, and her respiratory rate is 30 breaths/minute. She is disoriented, and her skin is warm and moist. Which of the following antibiotics should be administered to treat this patient’s condition?

Answer Choices Correct AnswerCorrect answer Your AnswerYour answer
A. Cefazolin
B. Cefotaxime
C. Clarithromycin
D. Metronidazole
E. Vancomycin
Option B (Cefotaxime) is correct. The management of febrile neutropenic patients requires expeditious establishment of the etiology and institution of broad-spectrum antibiotics as soon as possible. Neutropenic patients are at risk of Pseudomonas aeruginosa and other gram-negative rods and Staphylococcus aureus. The third-generation cephalosporin has a broad spectrum that covers these organisms and is usually the first-line treatment in combination with an aminoglycoside.

Option A (Cefazolin) is incorrect. First-generation cephalosporins cover gram-positive organisms well but miss many gram-negatives, particularly Pseudomonas.

Option C (Clarithromycin) is incorrect. Macrolides do not cover gram-negative organisms as well.

Option D (Metronidazole) is incorrect. Metronidazole is a good agent for anaerobic organisms but does not cover the other organisms well.

Option E (Vancomycin) is incorrect. Vancomycin is an excellent agent for gram-positive organisms but does not cover gram-negatives well. Use of vancomycin is also discouraged to prevent the development of resistant organisms.

High-yield Hit 1
Empirical antibiotic therapy
Intravenous antibiotics
Over 90% of the first episodes of infection in neutropenic cancer patients are caused by Gram-positive or Gram-negative bacteria (see Fig. 100.1). Empirical antibiotic regimens must be broad spectrum and bactericidal, achieve high circulating and tissue levels and be nontoxic. For more than two decades, combinations of two or more intravenous antibiotics have been the 'gold standard' of empirical antibiotic therapy. Numerous combinations of antipseudomonal penicillins (e.g. mezlocillin, ticarcillin with or without clavulanic acid, azlocillin or piperacillin) or third- or fourth-generation cephalosporins (ceftazidime, ceftriaxone, cefpirome, cefepime) plus an aminoglycoside (e.g. gentamicin, tobramycin, netilmicin or amikacin) have been frequently utilized.4,17,40,41 But no particular one was shown to be superior to the others. However, aminoglycoside-containing regimens are associated with renal and auditory toxicity, especially in patients concomitantly receiving other toxic agents, and often require monitoring of drug levels.
Thus, new treatment approaches have been explored to avoid these complications, such as once-daily dosing of aminoglycoside or β-lactam monotherapy. An international, multicenter trial by the EORTC-IATG showed that once-daily ceftriaxone and amikacin was as effective and at least no more toxic than a combination of ceftazidime and amikacin given thrice daily.42 Monotherapy with broad-spectrum and highly bactericidal agents, such as third- or fourth-generation cephalosporins (ceftazidime, cefepime, cefpirome), carbapenems (i.e. imipenem and meropenem) or antipseudomonal penicillins combined with a β-lactamase inhibitor (piperacillin/tazobactam), were found to be as effective as and less toxic than combinations of β-lactam and aminoglycoside.19,43-46 Choices of intravenous empirical antibiotic therapy for high-risk patients, typically those with hematological malignancies and long duration neutropenia, are shown in Figure 100.15.
Figure 100.15 Choices of empirical intravenous antibiotics in high-risk febrile neutropenic cancer patients.
Choices should be guided by the patient's clinical condition and local epidemiological data. Consider using an aminoglycoside-containing regimen in critically ill patients, such as those with severe sepsis or septic shock, when a P. aeruginosa infection is suspected or when resistant Gram-negative bacteria prevail.5,37 Consider using a glycopeptide antibiotic (i.e. vancomycin or teicoplanin) in patients with catheter-related infections, when penicillin-resistant streptococcal or methicillin-resistant staphylococcal infections are suspected or in critically ill patients (i.e. with severe sepsis or septic shock).5,37 However, outside these well-defined clinical circumstances encountered in a minority of patients, routine use of glycopeptide antibiotics is strongly discouraged. Empirical use of vancomycin has not been shown to improve patient outcome and is associated with increased costs, toxicity and emergence of resistance.

Taken from Infectious Diseases by Cohen.
Macrolide

A. Cefazolin
B. Cefotaxime
C. Clarithromycin
D. Metronidazole
E. Vancomycin
Option C (Clarithromycin)
First-generation cephalosporin

A. Cefazolin
B. Cefotaxime
C. Clarithromycin
D. Metronidazole
E. Vancomycin
Option A (Cefazolin)
First-generation cephalosporins cover gram-positive organisms well but miss many gram-negatives, particularly Pseudomonas.
third-generation cephalosporin

A. Cefazolin
B. Cefotaxime
C. Clarithromycin
D. Metronidazole
E. Vancomycin
Option B (Cefotaxime)
A 16-year-old boy has had a 5-day history of progressively worsening dry cough, malaise, fatigue, headache, and chills. He has also noticed a nonpruritic, nonpainful rash over his trunk. He was previously well, does not take any regular medications, and has never been hospitalized. His vital signs are blood pressure, 110/70 mm Hg; pulse, 101 beats/min; temperature 37.9°C (100.2°F); and respirations, 21 breaths/min. The lung fields are clear to auscultation bilaterally and there is a maculopapular, slightly erythematous rash over the trunk. A chest X-ray demonstrates extensive, streaky, interstitial infiltration and patches of atelectasis in the left lower lobe, along with a thickened bronchial shadow. Hematological investigation will most likely reveal what finding?

Answer Choices Correct AnswerCorrect answer Your AnswerYour answer
A. Cold agglutinins
B. Hemolytic anemia
C. Iron-deficiency anemia
D. Leukopenia
E. Thrombocytopenia
Option A (Cold agglutinins) is correct. This patient presents with clinical and radiographic evidence of an atypical pneumonia combined with a maculopapular rash. This suggests infection with Mycoplasma pneumoniae, one of the most common causes of atypical pneumonia that particularly affects adolescents. There are numerous extrapulmonary compilations of Mycoplasma infection, including hematologic. The most common abnormality is formation of cold agglutinins and is present in slightly more than half of the cases. This finding is used as part of the diagnosis of Mycoplasma but is somewhat nonspecific because cold agglutinins can be elevated in Epstein-Barr virus infections, cytomegalovirus infections, and other viral illnesses.

Option B (Hemolytic anemia) is incorrect. Subclinical hemolytic anemia is a common finding but less common than the development of cold agglutinins. Rarely, patients will be present with overt anemia. Any patient with pneumonia and laboratory evidence of a hemolytic anemia should be suspected of having Mycoplasma.

Option C (Iron-deficiency anemia) is incorrect. Iron-deficiency anemia is not commonly associated with Mycoplasma infection.

Option D (Leukopenia) is incorrect. In many cases of Mycoplasma infection, the white blood cell count is normal. The vast majority of the remaining cases show a leukocytosis. Leukopenia, however, is a rare finding in Mycoplasma infection.

Option E (Thrombocytopenia) is incorrect. Thrombocytosis is occasionally found in patients with Mycoplasma infection. Thrombocytopenia, however, is rare.

High-yield Hit 1
Mycoplasma pneumoniae

1. Identification
1. Slow-growing, obligate aerobe forming granular colonies on Eaton's agar
2. Serologic tests for antigen-specific IgM antibodies
2. Pathogenesis
1. P1 adhesin protein mediates adherence to respiratory epithelium.
2. Damage to ciliated epithelium by lytic enzymes and hydrogen peroxide (H2O2) leads to decreased clearance of upper airways, facilitating the spread of bacteria to the lungs.
3. Superantigen activity is related to systemic symptoms of M. pneumoniae infection.
3. Diseases caused by M. pneumoniae
* Mycoplasma pneumoniae is spread via aerosols from infected persons and asymptomatic carriers.
1. Primary atypical ("walking") pneumonia
* Comparatively mild interstitial pneumonia that differs clinically from lobar pneumonia caused by pneumococcus (Table 13-1)
* Initial malaise, low-grade fever, and headache followed after 2 to 4 days by nonproductive cough, rales, rhonchi, myalgia, and, rarely, maculopapular rash
2. Tracheobronchitis
* Inflammation of the bronchi marked by nonproductive cough, fever, headache, sore throat, pharyngeal exudates, and cervical lymphadenopathy
3. Pharyngitis
* May precede pneumonia or constitute milder presentation of M. pneumoniae infection
* Resembles nonexudative group A streptococcal or viral pharyngitis
4. Treatment
* Macrolides, tetracycline, or fluoroquinolones (e.g., levofloxacin). No target for β-lactam antibiotics

Table 13-1. Mycoplasmal versus Pneumococcal Pneumonia
Characteristic Mycoplasmal Pneumonia Pneumococcal Pneumonia
Preceding pharyngitis Common Never
Onset Gradual Rapid with chills
Fever Low grade High
Cough Nonproductive, paroxysmal Productive
Sputum Usually clear Blood-tinged and purulent
Pleuritic chest pain Absent Present
Empyema Absent Sometimes present
Leukocytosis Absent Usually present, although neutropenia is seen in severe cases
Age of highest incidence Young adults Older adults
Complications Otitis media, erythema multiforme, hemolytic anemia, myocarditis, pericarditis Bacteremia, meningitis, otitis media

From Rapid Review Microbiology & Immunology 2E by Rosenthal & Tan
Any patient with pneumonia and laboratory evidence of a hemolytic anemia should be suspected of having
Mycoplasma
This patient has the triad of fever, headache, and rash combined with tick exposure. She is also pregnant. Treat with:
A. Ampicillin
B. Azithromycin
C. Chloramphenicol
D. Doxycycline
E. Penicillin
Option C (Chloramphenicol)
Doxycycline cannot be given to pregnant woman, thus, the only available alternative is chloramphenicol.
often used in Lyme disease, which presents as an erythematous patch with central clearing. It is not used in the management of Rocky Mountain spotted fever (RMSF).

A. Ampicillin
B. Azithromycin
C. Chloramphenicol
D. Doxycycline
E. Penicillin
Option A (Ampicillin). Ampicillin is often used in Lyme disease, which presents as an erythematous patch with central clearing. It is not used in the management of Rocky Mountain spotted fever (RMSF).
the treatment of choice for community-acquired pneumonia, which would present with lobar consolidation or streaky infiltrates.

A. Azithromycin
B. Clindamycin
C. Isoniazid
D. Metronidazole
E. Trimethoprim-sulfamethoxazole
Option A (Azithromycin)
is indicated for treatment of latent tuberculosis, where there is a positive purified protein derivative (PPD) test but no signs or symptoms of tuberculosis (TB). TB can present with cavitary lesions, but does not usually result in particularly odorous sputum.

A. Azithromycin
B. Clindamycin
C. Isoniazid
D. Metronidazole
E. Trimethoprim-sulfamethoxazole
Option C (Isoniazid)
is used in the therapy of Pneumocystis carinii pneumonia, which can present as a cavitary lesion but does not produce particularly foul-smelling sputum.

A. Azithromycin
B. Clindamycin
C. Isoniazid
D. Metronidazole
E. Trimethoprim-sulfamethoxazole
Option E (Trimethoprim-sulfamethoxazole)
gives rises to invasive mucormycosis, that presents similarly to bacterial sinusitis. Erythema or cyanosis of the facial skin overlying the frontal sinuses is a clue to this rapidly evolving infection. Nasal ulceration is another common clue. It is most common in diabetics or in immunocompromised individuals.

A. Escherichia coli
B. Moraxella catarrhalis
C. Rhizopus oryzae
D. Staphylococcus aureus
E. Streptococcus pneumoniae
Option C (Rhizopus oryzae)
is a very rare cause of community-acquired bacterial sinusitis. It typically occurs in patients located in the intensive care unit.

A. Escherichia coli
B. Moraxella catarrhalis
C. Rhizopus oryzae
D. Staphylococcus aureus
E. Streptococcus pneumoniae
Option A (Escherichia coli)
This is a common cause of sinusitis in children, accounting for up to 25% of cases. It is a much less common cause in adults, however.

A. Escherichia coli
B. Moraxella catarrhalis
C. Rhizopus oryzae
D. Staphylococcus aureus
E. Streptococcus pneumoniae
Option B (Moraxella catarrhalis)
Gram stain of the sputum usually shows a scarcity of organisms. Patients tend to present with a flulike illness and a nonproductive cough. The chest x-ray usually shows a patchy infiltrate. It is identified serologically by a direct fluorescent antibody test or by a urine antigen test.

A. Histoplasmosis capsulatum
B. Klebsiella pneumoniae
C. Legionella pneumophila
D. Mycobacterium tuberculosis
E. Pneumocystis carinii
Option C (Legionella pneumophila)
the classic finding on radiography is diffuse central (perihilar) alveolar or interstitial infiltrates. Typically there is little or no sputum production.

A. Histoplasmosis capsulatum
B. Klebsiella pneumoniae
C. Legionella pneumophila
D. Mycobacterium tuberculosis
E. Pneumocystis carinii
Option E (Pneumocystis carinii)
A 3-year-old girl is seen by her physician for an itchy rash. The girl has a “slapped cheek"” rash that the resident has seen many times before. She also has a lacy, reticular, maculopapular rash involving the trunk and extremities. The mother says that her pregnant sister is coming for a visit tomorrow and asks the resident if the girl’s condition could affect the sister. The mother has two other young children at home who have no similar symptoms. Which of the following is the most appropriate response?

Answer Choices Correct AnswerCorrect answer Your AnswerYour answer
A. Only children with underlying disease are susceptible
B. The child is contagious and the sister should not visit
C. The child is not contagious but the sister should not visit
D. The sister is at risk of contagion, but there is no associated risk to her unborn child
E. There is no risk because the child is not contagious at the time the rash is present
Option C (The child is not contagious but the sister should not visit) is correct. Erythema infectiosum, or fifth disease, is caused by parvovirus B19 infection and generally affects children 3 to 12 years of age. Children with fifth disease are not contagious at the time the rash is present. The mother’s other children may be asymptomatically infected and contagious even if they are showing no symptoms. Forty percent of adults are susceptible. Maternal infection can cause serious birth defects in the fetus.

Option A (Only children with underlying disease are susceptible) is incorrect. All children are susceptible to fifth disease. However, children with underlying disease, such as sickle cell disease or spherocytosis, may be at risk for aplastic crisis with parvovirus B19 infection.

Option B (The child is contagious and the sister should not visit) is incorrect. Fifth disease is mildly contagious. Children with fifth disease are not contagious at the time the rash is present. Forty percent of adults are susceptible. Maternal infection can cause serious birth defects in the fetus. The mother’s other children may be asymptomatically infected and contagious.

Option D (The sister is at risk of contagion, but there is no associated risk to her unborn child) is incorrect. The mother’s other children may be asymptomatically infected and contagious. Forty percent of adults are susceptible. Maternal infection can cause serious birth defects in the fetus.

Option E (There is no risk because the child is not contagious at the time the rash is present) is incorrect. Fifth disease is mildly contagious. Children with fifth disease are not contagious at the time the rash is present. The mother’s other children may be actively infected and contagious. Forty percent of adults are susceptible. Maternal infection can cause serious birth defects in the fetus.

High-yield Hit 1
ERYTHEMA INFECTIOSUM (FIFTH DISEASE).
The most common manifestation of parvovirus B19 is erythema infectiosum, also known as fifth disease, which is a benign, self-limited exanthematous illness of childhood. It was the fifth in a classification scheme of childhood exanthems. The preceding four exanthems were measles, scarlet fever, rubella and Filatov-Dukes disease (an atypical scarlet fever), with roseola infantum as the "sixth disease."
The incubation period for erythema infectiosum ranges from 4 to 28 days (average, 16-17 days). The prodromal phase is mild and consists of low-grade fever, headache, and symptoms of mild upper respiratory tract infection. The hallmark of erythema infectiosum is the characteristic rash, which occurs in three stages that are not always distinguishable. The initial stage is an erythematous facial flushing, often described as a "slapped-cheek" appearance. The rash spreads rapidly or concurrently to the trunk and proximal extremities as a diffuse macular erythema in the second stage. Central clearing of macular lesions occurs promptly, giving the rash a lacy, reticulated appearance. Palms and soles are spared, and the rash tends to be more prominent on extensor surfaces. Affected children are afebrile and not ill-appearing. Older children and adults often complain of mild pruritus. The rash resolves spontaneously without desquamation but tends to wax and wane over 1-3 wk. It can recur with exposure to sunlight, heat, exercise, and stress. Lymphadenopathy and atypical papular, purpuric, vesicular rashes are also described.

Taken from Nelson Textbook of Pediatrics by Behrman.
High-yield Hit 2
IMMUNOCOMPROMISED PERSONS.
page 1049
0
page 1050
Patients with impaired humoral immunity are at risk for chronic infections with parvovirus B19. Chronic anemia is the most common manifestation, sometimes accompanied by neutropenia, thrombocytopenia, or complete marrow suppression. Chronic infections are seen in children with cancer receiving cytotoxic chemotherapy, children with congenital immunodeficiencies, patients on immunosuppressive therapy for transplants, children (and adults) with AIDS, and patients with functional defects in IgG production who are unable to generate neutralizing antibodies.
FETAL INFECTION.
Primary maternal infection is associated with nonimmune fetal hydrops and intrauterine fetal demise; the risk of fetal loss after infection is estimated at <5%. The mechanism of fetal disease appears to be a viral-induced red cell aplasia at a time when the fetal erythroid fraction is rapidly expanding. This can lead to profound anemia, high-output cardiac failure, and hydrops. Viral DNA has been detected in infected abortuses. The second trimester seems to be the most sensitive period, but fetal losses are reported at every stage of gestation. Most infants infected in utero are born normally at term, even some with ultrasonographic evidence of hydrops. Some of these infants may acquire a chronic or persistent postnatal infection with B19, but its significance is unknown. Congenital anemia associated with intrauterine B19 infection has been reported in a few cases, sometimes following intrauterine hydrops. This process may mimic other forms of congenital hypoplastic anemia (i.e., Diamond-Blackfan syndrome). Fetal infection with B19 has not been associated with other birth defects. B19 is only one of many causes of hydrops fetalis (Chapter 92).

Taken from Nelson Textbook of Pediatrics by Behrman.